Примеры по алгебре 7 класс решать: 7 класс, самостоятельные работы по Мордковичу. Примеры за 1, 2, 3, 4 четверть

Содержание

Как научиться решать примеры по алгебре намного быстрее?

В седьмом классе уже нет обычной математики: она делится на более сложные предметы — алгебру и геометрию. И именно на этом этапе у многих учеников начинаются серьезные сложности с обучением. Это связано с тем, что появляется много новых, непростых тем. В 7 классе ученики изучают математическую модель, линейную функцию, степень с натуральным показателем, одночлены, многочлены и многое другое. Домашние задания, как и упражнения в классе, становятся больше, объемнее. Если допустить одну маленькую ошибку, то ее исправление нередко занимает полчаса и больше.

Вот почему гдз по алгебре 7 класс мордкович — пособие, которое всегда нужно иметь под руками. Это решебник, благодаря которому ученик в любой момент проверит то, что он написал, и сможет обнаружить ошибку, не тратя на это много времени. ГДЗ подходят для самых разных задач — подготовки домашнего задания, работы в классе, повторения всего материала перед контрольными работами.

Чем дальше идет обучение в седьмом классе, тем полезнее решебник, потому что запомнить большой объем информации очень трудно — и надежная «шпаргалка» никогда не помешает.

Чем удобны пособия по алгебре

Если посоветоваться с учителем можно только в классе, то решебник всегда под рукой — и там уже есть правильные ответы на все вопросы. А еще многие примеры предполагают получение ответа разными способами. Обычно ученику приходит в голову только один способ, а до остальных догадаться трудно. Как раз решебник и подскажет, как еще можно добиться того же результата, но другим путем — это очень полезно для развития мозга и для последующего обучения.

Онлайн-формат обучения еще больше плюсов

На сайте Випгдз все пособия представлены в онлайн-формате, и у этого решения сразу несколько плюсов:

  • Максимальная экономия места в рюкзаке и на книжных полках. Решебники по всем предметам остаются в телефоне или компьютере — для доступа к ним достаточно интернета и подходящего устройства.
  • Удобство поиска упражнений. Задачи и примеры пронумерованы с учетом того, как они располагаются в учебнике. Все находится за считаные минуты, а то и секунды.
  • Возможность использовать пособия в школе. Есть учителя, которые не очень жалуют ГДЗ, а потому запрещают брать такие книги в школу. Но если решебник находится в телефоне, он никогда не попадется учителю на глаза.

«Випгдз» — это хорошая помощь для тех, кто хочет отлично справляться с домашними заданиями, повышать уровень знаний и решать контрольные на высокие оценки. Если тщательно изучать тему, тренироваться и параллельно проверять свои ответы в ГДЗ, каждый урок принесет еще больше пользы.

Линейные уравнения 7 класс | Алгебра

Линейные уравнения, решение которых начинается в курсе алгебры (7 класс) — это уравнения вида

   

где a и b — числа, x — переменная.

Уравнения, сводящиеся к виду ax=b при помощи раскрытия скобок, приведения подобных слагаемых, переноса слагаемых из одной части уравнения в другую, а также умножения или деления обеих частей на число, отличное от нуля (то есть при помощи равносильных преобразований), также часто называют линейными (правильнее называть их уравнениями, сводящимися к линейным).

Рассмотрим примеры уравнений, сводящихся к линейным, которые встречаются в начале курса алгебры 7 класса.

   

Раскрываем скобки. Если перед скобками стоит множитель, умножаем этот множитель на каждое слагаемое в скобках. Если перед скобками стоит знак «+», знаки  не меняем. Если перед скобками стоит знак «-«, знаки меняем на противоположные:

   

Неизвестные слагаемые переносим в одну сторону, известные — в другую. При переносе знаки слагаемых меняем на противоположные:

   

   

Обе части уравнения делим на число, стоящее перед иксом:

   

   

Ответ: -9.

   

Раскрываем скобки:

   

Неизвестные слагаемые перенесём в левую часть, известные — в правую. Знак каждого слагаемого при переносе из одной части уравнения в другую меняем на противоположный:

   

(Обратите внимание: хотя сумма слагаемых  с переменной равна нулю, результат записываем не как 0, а как 0x).

Какое бы число мы не подставили в это уравнение вместо x, получим верное равенство.

Ответ: x — любое число.

   

Раскрываем скобки:

   

Можно сначала привести подобные слагаемые, чтобы упростить уравнение:

   

а уже потом перенести: неизвестные — в одну сторону, известные — в другую:

   

   

Это уравнение не имеет корней.

Ответ: нет корней.

   

Раскрываем скобки:

   

Приводим подобные слагаемые:

   

Переносим неизвестные слагаемые в одну сторону, известные — в другую, изменив при этом их знаки:

   

   

Обе части уравнения делим на число, стоящее перед иксом:

   

Ответ:

   

В следующий раз рассмотрим сводящиеся к линейным уравнениям уравнения с дробями.

Примеры дидактических игр на уроках АЛГЕБРЫ в VII КЛАССЕ.

«Многочлены и действия над ними»

МОУ Падовская СОШ Урок обобщения и повторения по теме «Многочлены и действия над ними» Алгебра 7 класс. Учитель:Кривоножкина И.Н. «Многочлены и действия над ними». Алгебра. 7-Й класс Цели: обобщение и

Подробнее

ISBN К 22.14я721 ISBN

ДК 373:512 К 22.14721 49 49 аа, аьяа Маа.. 7 9 /.М.. М : Э, 2018. 128. (. ). ISBN 978-5-04-093533-8, 7 9-. П ё -. П,. П 7 9-,, -. ДК 373:512 К 22.14я721 ISBN 978-5-04-093533-8 аа.м., 2018 О. ООО «Иаь «Э»,

Подробнее

Математика 8 класс Многочлены

МИНИСТЕРСТВО ОБРАЗОВАНИЯ И НАУКИ РОССИЙСКОЙ ФЕДЕРАЦИИ НОВОСИБИРСКИЙ ГОСУДАРСТВЕННЫЙ УНИВЕРСИТЕТ СПЕЦИАЛИЗИРОВАННЫЙ УЧЕБНО-НАУЧНЫЙ ЦЕНТР Математика 8 класс Многочлены Новосибирск Многочлены Рациональными

Подробнее

РАБОЧАЯ ПРОГРАММА ПО АЛГЕБРЕ ДЛЯ 7 класса

РАБОЧАЯ ПРОГРАММА ПО АЛГЕБРЕ ДЛЯ 7 класса Пояснительная записка Рабочая программа учебного курса по алгебре для 7 класса разна на основе Примерной программы основного общего образования (базовый уровень)

Подробнее

МАТЕМАТИКА. Квадратные корни

МАТЕМАТИКА Квадратные корни Задание для 8-х классов (006-00 учебный год) 4 Введение Дорогие ребята! Вы получили очередное задание по математике. В этом задании мы знакомим вас с важным математическим понятием

Подробнее

Учебник: Алгебра (Дорофеев Г.В.) гг.

Класс 9.3.1, 9.3.2 (база) Учебник: Алгебра (Дорофеев Г.В.) 2018-2019 гг. Тема модуля: «Уравнения и системы уравнений» Основные теоретические сведения, необходимые для успешного выполнения теста: 1. Понятие

Подробнее

КАЛЕНДАРНО — ТЕМАТИЧЕСКОЕ ПЛАНИРОВАНИЕ

КАЛЕНДАРНО — ТЕМАТИЧЕСКОЕ ПЛАНИРОВАНИЕ урока тема урока 1 Повторение за курс 6 класса. Решение 2 Повторение за курс 6 класса. Положительные и отрицательные числа. 3 Повторение за курс 6 класса. Пропорции.

Подробнее

Итоговый тест по алгебре 7 класс

Итоговый тест по алгебре 7 класс Вариант 1 Базовый уровень А1. Найдите значение выражения: 7,8 6,3+7,8 13,7 1)156 2)78 3)-78 4) 146. А2. Решите уравнение: 5у-3,5=2у+5,5 1)5. 2)-3. 3) 3. 4) 4. А3. Упростите

Подробнее

МАТЕМАТИЧЕСКИЙ ПРАКТИКУМ

МАТЕМАТИЧЕСКИЙ ПРАКТИКУМ Ю.Л.Калиновский Введение Решение квадратных уравнений Решение квадратных уравнений c помощью разложения на множители. Решение квадратных уравнений c помощью дополнения до полного

Подробнее

Тематическое планирование

Количество часов Наименование разделов и тем Тематическое планирование Планируемые образовательные результаты 1 Линейное уравнение с одной переменной 2 Целые выражения 15 Распознавать числовые выражения

Подробнее

Дробно-рациональные выражения

Дробно-рациональные выражения Выражения содержащие деление на выражение с переменными называются дробными (дробно-рациональными) выражениями Дробные выражения при некоторых значениях переменных не имеют

Подробнее

8 класс Алгебра. Тема «Рациональные дроби»

8 класс Алгебра Тема «Рациональные дроби» 1. ОСНОВНЫЕ ПОНЯТИЯ Понятие алгебраической дроби знакомо вам из курса алгебры 7-го класса, где мы довольно много внимания уделили сокращению алгебраических дробей.

Подробнее

ПОЯСНИТЕЛЬНАЯ ЗАПИСКА

ПОЯСНИТЕЛЬНАЯ ЗАПИСКА Рабочая программа по математике для 7 класса составлена на основе следующих документов: 1. Федеральный компонент государственного стандарта основного общего образования по математике.

Подробнее

Образовательный минимум по алгебре

Приложение 2 Тема: Алгебраические выражения Образовательный минимум по алгебре Порядок выполнения арифметических действий Сложение и вычитание — действия первой ступени Умножение и деление — действия второй

Подробнее

УРАВНЕНИЯ В ЦЕЛЫХ ЧИСЛАХ

УРАВНЕНИЯ В ЦЕЛЫХ ЧИСЛАХ 1. Разложение на множители a) ( x 1)( y+ ) 9. б) x(y 98). в) x + y= xy. г) x + 4xy 7y. д) 19x yz 995, решить в простых числах. Делимость чисел а) y = 5x + 6. б) в) г) д) x + 1=

Подробнее

Пояснительная записка

Пояснительная записка Рабочая программа учебного курса алгебры для 7 класса составлена на основе Федерального государственного образовательного стандарта основного общего образования второго поколения,

Подробнее

Е. Н. ФИЛАТОВ АЛГЕБРА

Заочный физико-математический лицей «Авангард» Е. Н. ФИЛАТОВ АЛГЕБРА 8 Экспериментальный учебник Часть МОСКВА 06 Заочный физико-математический лицей «Авангард» Е. Н. Филатов АЛГЕБРА 8 Экспериментальный

Подробнее

Функции и их графики Что такое функция п.12, 258, 259,261 Вычисление значений функции по формуле п.13, 267, 269, 271,272, 277

Тематическое планирование по математике (экстернат) на 2017-2018 учебный год в 7 классе Учебник: Ю.И. Макарычев и др., Алгебра 7 класс, Просвещение, 2015 г и далее Дидактические материалы: Л.И. Звавич

Подробнее

1 Решение квадратных уравнений

Введение Решение квадратных уравнений Решение квадратных уравнений c помощью разложения на множители. Решение квадратных уравнений c помощью дополнения до полного квадрата Решение квадратных уравнений

Подробнее

Пояснительная записка

Пояснительная записка Сведения о программе Авторская программа «Программа для общеобразовательных учреждений. Планирование учебного материала. Алгебра. 7 9 классы» / авт.-сост. И. Е. Феоктистов. М.: Мнемозина,

Подробнее

ПОЯСНИТЕЛЬНАЯ ЗАПИСКА

1 ПОЯСНИТЕЛЬНАЯ ЗАПИСКА Рабочая программа составлена на основании следующих документов: Федеральный компонент государственного образовательного стандарта начального общего, основного общего и среднего

Подробнее

УДК :512 ISBN УДК :512. ББК я721

УДК 373.167.1:51 ББК.141я71 М5 Мерз ляк А.Г. М5 Ал геб ра : 7 класс : са мо стоя тель ные и кон троль ные рабо ты : по со бие для уча щих ся об ще об ра зо ва тель ных организаций / А.Г. Мерз ляк, В.Б.

Подробнее

РАБОЧАЯ ПРОГРАММА по математике 7 класс

Муниципальное автономное общеобразовательное учреждение «Средняя общеобразовательная школа 76»» Программа согласована на заседании методического совета МАОУ «СОШ 76», протокол 1 от 28.08.2015г РАБОЧАЯ

Подробнее

Пояснительная записка

Пояснительная записка Данная рабочая программа ориентирована на учащихся 7 класса и реализуется на основе следующих документов: 1. Государственный стандарт начального общего, основного общего и среднего

Подробнее

КАЛЕНДАРНО-ТЕМАТИЧЕСКОЕ ПЛАНИРОВАНИЕ

КАЛЕНДАРНО-ТЕМАТИЧЕСКОЕ ПЛАНИРОВАНИЕ АЛГЕБРА 7 КЛАСС К учебнику Макарычев Ю.Н., Миндюк Н.Г., Нешков К.И., Суворова С.Б. Всего 102 часа (3 часа в неделю) урока Тема урока 1. Числовые выражения Колво часо

Подробнее

Рабочая программа по алгебре 8 А класса

Государственное бюджетное общеобразовательное учреждение Республики Хакасия «Хакасская национальная гимназия интернат им. Н.Ф.Катанова» «СОГЛАСОВАНО» на заседании кафедры математики и информатики Протокол

Подробнее

▶▷▶▷ гдз решить уравнение к

▶▷▶▷ гдз решить уравнение к
ИнтерфейсРусский/Английский
Тип лицензияFree
Кол-во просмотров257
Кол-во загрузок132 раз
Обновление:05-10-2019

гдз решить уравнение к — Гдз Решить Уравнение К — Image Results More Гдз Решить Уравнение К images Калькулятор онлайн — Решение показательных уравнений wwwmath-solutionrumath-taskexponential-equality Cached Решить уравнение 9 х — 4 3 х — 45 0 Заменой 3 х t данное уравнение сводится к квадратному уравнению t 2 — 4t — 45 0 Решая это уравнение , находим его корни: t 1 9, t 2 -5, откуда 3 х 9, 3 х -5 Решить уравнение — pocketteacherru pocketteacherru Cached Pocket Teacher: здесь вы можете решить уравнение и математические задачи любой сложности за секунды совершенно бесплатно 5 Решите уравнение 7 класс — YouTube wwwyoutubecom watch?vLWKXlsudNuI Cached Решенные примеры по алгебре 7 класс https: wwwyoutubecom playlist?listPLCrA1G_tvIUtZe5f-wgB7xxvH-C0OgdYJ по учебнику Решите уравнение ОГЭ по математике Задание 21 вариант 3 wwwyoutubecom watch?vO1_A27gO45c Cached Решите уравнение огэ задание 21 вариант 3 Подготовка к огэ по математике под редакцией Ященко 2018 Как решить Решение уравнений Математика, 6 класс: уроки, тесты, задания wwwyaklassrupmatematika6-klass Cached Теоретические уроки, тесты и задания по предмету Решение уравнений, Преобразование буквенных выражений, 6 класс, Математика ГДЗ: готовые домашние задания за 1-11 класс — GDZRU gdzru Cached Вы выбрали отличный сайт с гдз , где собрана вся информация по решению домашнего задания Давно не секрет, что школьники постоянно пользуются решебниками к школьным учебникам Решение уравнений Учебник по математике 6 класс (Виленкин) urokitvreshenie-uravnenij-uchebnik-po Cached Зная правила нахождения неизвестного слагаемого, Вы уже можете решать уравнения вида х35 Вы знаете, что х35, х5-32 Легко! А если есть такое уравнение 3х520, как его решить ? ГДЗ — готовые домашние задания и решебники megareshebaru Cached ГДЗ (готовые домашние задания) за 1-11 классы, онлайн решебники по всем школьным предметам, ответы к рабочим и лабораторным тетрадям Уравнения 5 класса Математика wwwfor6cluznatesheruuravneniya-5-klassa Cached Как решить вот это уравнение ? (54у 83) 21 231 Я считаю что там где стоит вычитание должно быть умножение Задание на печатала сама учительница, поэтому всё должно быть правильно 10 Уравнение — ГДЗ учебник — matem-gdzru matem-gdzru5-klassotvety-gdz-k-uchebniku-po Cached Главная 5 класс Ответы ( ГДЗ ) к учебнику по математике 5 класс Решить уравнение (у 64) — 38 Promotional Results For You Free Download Mozilla Firefox Web Browser wwwmozillaorg Download Firefox — the faster, smarter, easier way to browse the web and all of 1 2 3 4 5 Next 27,300

  • …469; в) 2у 7у 78 1581; г) 256m — 147m — 1871 — 63 747; д) 88 880: 110 х 809; е) 6871 р: 121
  • 7000; ж) 3810 1206: у 3877; з)… Готовые домашние задания ГДЗ по Математике автор Зубарева, Мордкович за 6 класс. Решать уравнения. Поэтому вам не составит труда быстро найти и решить определённ
  • рдкович за 6 класс. Решать уравнения. Поэтому вам не составит труда быстро найти и решить определённый номер. Данный сайт можно использовать для получения ГДЗ по алгебре за 7 класс, как решебник для решения задач по алгебре онлайн. Не надо ничего списывать, всё здесь. Подготовка к ЕГЭ по математике, варианты, тесты, конспекты по математике, алгебре, геометрии. Степенное алгебраическое уравнение. ax 2 n bx n c 0 (a ne 0) Заменой y x n приведем к уравнению… Практическая польза от ГДЗ по алгебре 7 класс Специальное пособие, которое можно просмотреть онлайн и скачать, ГДЗ по алгебре 7 класс Iстер (Истер) позволит не просто освоить данный предмет, но также и решить все проблемы, возникшие в ходе образовательного процесса. Практически, при определенных навыках и умениях, все решается … ГДЗ по математике за 5 класс Виленкин Н.Я. ГДЗ Виленкин В.Я. Материал удален по требованию издательства quot;Экзаменquot; Где скачать ГДЗ по английскому 5-6 класс Биболетова рабочая тетрадь??? Если также будет просить кого-либо решить за тебя такие элементарные задачи, то к концу года забудешь 22. Контрольные Олимпиады ЕГЭ ГДЗ. Любое уравнение можно привести к виду f(x) 0 и считать уравнением частный случай функции у f(x), когда она равна нулю. ГДЗ Алгебра 7 Колягин, Ткачева, Федорова 510. Задачи для ОГЭ с ответами и решениями Линейные уравнения перейти к содержанию задачника Решите уравнение.

Ткачева

как решебник для решения задач по алгебре онлайн. Не надо ничего списывать

  • 3 х -5 Решить уравнение — pocketteacherru pocketteacherru Cached Pocket Teacher: здесь вы можете решить уравнение и математические задачи любой сложности за секунды совершенно бесплатно 5 Решите уравнение 7 класс — YouTube wwwyoutubecom watch?vLWKXlsudNuI Cached Решенные примеры по алгебре 7 класс https: wwwyoutubecom playlist?listPLCrA1G_tvIUtZe5f-wgB7xxvH-C0OgdYJ по учебнику Решите уравнение ОГЭ по математике Задание 21 вариант 3 wwwyoutubecom watch?vO1_A27gO45c Cached Решите уравнение огэ задание 21 вариант 3 Подготовка к огэ по математике под редакцией Ященко 2018 Как решить Решение уравнений Математика
  • поэтому всё должно быть правильно 10 Уравнение — ГДЗ учебник — matem-gdzru matem-gdzru5-klassotvety-gdz-k-uchebniku-po Cached Главная 5 класс Ответы ( ГДЗ ) к учебнику по математике 5 класс Решить уравнение (у 64) — 38 Promotional Results For You Free Download Mozilla Firefox Web Browser wwwmozillaorg Download Firefox — the faster
  • тесты

Нажмите здесь , если переадресация не будет выполнена в течение нескольких секунд гдз решить уравнение к Поиск в Все Картинки Ещё Видео Новости Покупки Карты Книги Все продукты Решение уравнений бесплатно Калькулятор Онлайн kontrolnayarabotaruequ Решение уравнений бесплатно Онлайн калькулятор для решения любых уравнений , неравенств, интегралов Обычные уравнения Дифференциальные Квадратные уравнения Решите уравнение Решебник ГДЗ по математике ЕГЭ и egegiablogspotcomxxhtml?m Ярлыки Разложите многочлен на множители, Разложите на множителе, Решите уравнение Приложения в Play Photomath Рейтинг , голосов Бесплатно Android Обучение Научитесь решать математические задачи, проверять домашние задания и готовиться к предстоящим Задача ГДЗ решебник Математика класс Мерзляк mathcomua gdz merzlyak ГДЗ по математике класс Мерзляк Задача Решите уравнение ,x ,x , ,; a a Подскажите решение уравнения Поделитесь гдз gdz podskazhite Ответы на вопрос Подскажите решение уравнения Поделитесь гдз алгебра класс Мордкович? Как решить уравнение в Помогите с гдз по алгебре gdz kakreshit Ответы на вопрос Как решить уравнение в Помогите с гдз по алгебре класс Мордкович? читайте на ГДЗ Математика класс Тарасенкова Уравнения reshebnikiuchebnikiru uravneniya ph ГДЗ , решебник Математика класс Тарасенкова Уравнения Задание Можно ли решить уравнение Задача Математика класс решебник гдз Vcevceru vcevcerumvn Решите уравнение а х ; в а ; б х ; г b Решение уравнений класс Школьный помощник Математика класс Решение уравнений Правила Задания с проверкой ответов Математика класс Решение задач на составление июл Решение задач на составление уравнений Образование Обучение Znaika TV Знайкару myoutubecom ГДЗ к Задачнику по Алгебре за класс АГ Мордкович и др gdz algebraag Укажите промежуток, которому принадлежит корень уравнения x x х , Решите уравнение Страница ГДЗ класс, Математика, Моро, Волкова gdz taskview Зная это, можно решать уравнения , в которых неизвестным является уменьшаемое Объясни решение Номер ГДЗ по Математике класс Виленкин НЯ https gdz putinainfonomer Условие Решите уравнение а х х ,; в а а , ; б y у ,; г k k , , РешеноУпр ГДЗ Алимов класс по алгебре Решить уравнение sinx ; cosx; xinx sinx; cosxcos Решение уравнений Учебник по математике класс gdz netreshenieuravnenij Решебник Математика класс Виленкин Легко! А если есть такое уравнение х, как его решить ? Решите уравнение ГДЗ и решебники от VIPGDZcom gdz comuprazhnenie Подробный ответ из решебника ГДЗ на Упражнение по учебнику АГ Мерзляк, ВБ Полонский, МС Якир класс уравнения Как решить уравнение , примеры uravneni Задачи во классе уравнения , как решить уравнение Задания по математике Задача по математике класс Виленкин wwwkakkaknet?viewsolve_vil Решить уравнения а х , , ,; в ,х , ,; б , х , ; г , х , , Задача по математике класс Виленкин kakkaknet kakkaknet?viewsolve_vilnumb Решите уравнение а ,x ,x , ,; в , , ,; б ,у , у , ,; г , m , Уравнение Ответы ГДЗ рабочая тетрадь Мерзляк gdz ru gdz Решить уравнение значит найти все его корни или показать, что их нет Чтобы найти неизвестное Уравнения класса Математика wwwforcluznatesheru uravneniya Привет помоги пожалуйста решить уравнение ,x, Добрый день, Марианна! Да, ответы с остатком ГДЗ по , Готовое домашнее задание mygdzcom wwwmy gdz comalgebraklass Наш робот распознал Решите уравнение а Зхх ; д у у I ; б хх Э ; е хх ; Решите уравнение gdz foxru gdz foxrureshiteuravneni ГДЗ к учебнику по алгебре для класса Макарычев ЮН и др Упражнение Ответ Уравнения с модулем Подготовка к ЕГЭ по математике uravneniya s Необходимо решить уравнение вида Чтобы не теряться в таких случаях, научимся решать уравнения с модулем все Важно проследить, чтобы ответы соответствовали интервалу! Помогите пожалуйста решить гдз по Алгебре Решите апр Помогите пожалуйста решить гдз по Алгебре Решите уравнение хy xy График Решение задач онлайн Math mathcomreshenie Решить ваши задачи онлайн с нашей программой Бесплатная версия программы предоставляет Вам только ответы раздела математики а также координатных задач, простых уравнений , неравенств, упрощать выражения Решебник по математике, решение примеров и задач mateshkaru Что решебник может решить Данная программа Системы дифференциальных уравнений , xy, xy ЛовиОтвет Решебник и калькулятор с решениями loviotvetru Калькулятор ЛовиОтвет решает математические примеры и уравнения с отображением этапов решения, Решение уравнений с модулем Математика, которая мне hijosrureshenieuravnenijs Решение уравнений с модулем Решить уравнение мои ответы ; и в учебнике другие Калькулятор онлайн Решение показательных уравнений mathsolutionru Этот математический калькулятор онлайн поможет вам решить показательное уравнение Программа для Тема Уравнения и системы уравнений Материалы для Решите уравнение х х Решите уравнение х х х х Ответы ; ; ; ; ; ; ; ; ; ; ; ; ; ; ; математика Решим всё Решите уравнение просмотры математика класс Решите дифференциальное уравнение yyxxex Тригонометрические уравнения , исследование Решу ЕГЭ Задание а Решите уравнение б Найдите все корни этого уравнения , принадлежащие отрезку Линейные уравнения Решение, примеры egesdamrupagephp авг Что такое линейные уравнения ? Как они Пусть нам нужно решить вот такое уравнение х х В математике такие ответы частенько встречаются Вот так Сейчас ГДЗ по математике для класса НЯ Виленкин номер gdz matematika Решите уравнение а у , , Решебник номер Решите уравнение а у , , Картинки по запросу гдз решить уравнение к App Store Photomath Apple Рейтинг , отзывов Бесплатно iOS Обучение Научитесь решать математические задачи, проверять домашние задания и готовиться к предстоящим задачи или введите и отредактируйте уравнения в нашем научном калькуляторе Математика класс Мерзляк, Полонский, Якир Номер Решите уравнение x ; x , ; y ; y , ,; m ; Гдз Знатоки гдз вопросов по теме Гдз Составим уравнение хх, Почему решение задач на ГДЗ стало платным? А вообще лучше решать задачи самому, так впоследствии легче учиться Решение уравнений Социальная сеть работников апр Самостоятельная работа содержит четыре варианта Первый и второй варианты Решение уравнений онлайн Онлайнкалькулятор Решение квадратных, кубических, тригонометрических, логарифмических уравнений онлайн Нахождение лучших приложений для безупречной учебы в школе admeruluchshih Калькулятор может решить все типы уравнений кроме дифференциальных , исправляет ошибки в выражениях и Операции с дробями, онлайн калькулятор для решения webmathruprog_ph ввода данных, ответы на часто задаваемые вопросы и оставить свой комментарий Вы поняли, как решать ? С Решение уравнений Еуроки eurokiorg gdz Решебник по алгебре за класс авторы Звавич, Кузнецова издательство Просвещение Задание С Решение Номер гдз по математике класс Муравин, Муравина https gdz plusmeklasszadacha Задача , ГДЗ по математике за класс к учебнику Муравина Бесплатные ответы Решение уравнений Решите уравнение а х х х ; б х https gdz expertmordcovich ГДЗ по алгебре за класс Мордковича Упражнения решение задания Решите уравнение а х х х Решение дифференциальных уравнений из сборника решурфматематикаФилиппов Бесплатный решебник для задачника АФ Филиппова Самая полная база решений, свободный доступ без Дискриминант Решение квадратных уравнений через mathprostoru?pagepagesof По одной из версий термин Дискриминант произошел от латинского discriminantis, что означает Линейное уравнение с одной переменной й класс открытыйурокрфarticles Обобщение умения решать уравнения сведением их к линейному уравнению мин Коллективная работа с Решение простейших линейных уравнений Павел Бердов berdovcomprosteyshie окт В этом видео мы разберём целый комплект линейных уравнений , которые решаются по одному Запросы, похожие на гдз решить уравнение к пошаговое решение уравнений онлайн решение уравнений класс решение уравнений со степенями решение уравнений класс решить уравнение с дробями решение уравнений онлайн по фото как решить уравнение класс решение уравнений класс След Войти Версия Поиска Мобильная Полная Конфиденциальность Условия Настройки Отзыв Справка

…469; в) 2у 7у 78 1581; г) 256m — 147m — 1871 — 63 747; д) 88 880: 110 х 809; е) 6871 р: 121 7000; ж) 3810 1206: у 3877; з)… Готовые домашние задания ГДЗ по Математике автор Зубарева, Мордкович за 6 класс. Решать уравнения. Поэтому вам не составит труда быстро найти и решить определённый номер. Данный сайт можно использовать для получения ГДЗ по алгебре за 7 класс, как решебник для решения задач по алгебре онлайн. Не надо ничего списывать, всё здесь. Подготовка к ЕГЭ по математике, варианты, тесты, конспекты по математике, алгебре, геометрии. Степенное алгебраическое уравнение. ax 2 n bx n c 0 (a ne 0) Заменой y x n приведем к уравнению… Практическая польза от ГДЗ по алгебре 7 класс Специальное пособие, которое можно просмотреть онлайн и скачать, ГДЗ по алгебре 7 класс Iстер (Истер) позволит не просто освоить данный предмет, но также и решить все проблемы, возникшие в ходе образовательного процесса. Практически, при определенных навыках и умениях, все решается … ГДЗ по математике за 5 класс Виленкин Н.Я. ГДЗ Виленкин В.Я. Материал удален по требованию издательства quot;Экзаменquot; Где скачать ГДЗ по английскому 5-6 класс Биболетова рабочая тетрадь??? Если также будет просить кого-либо решить за тебя такие элементарные задачи, то к концу года забудешь 22. Контрольные Олимпиады ЕГЭ ГДЗ. Любое уравнение можно привести к виду f(x) 0 и считать уравнением частный случай функции у f(x), когда она равна нулю. ГДЗ Алгебра 7 Колягин, Ткачева, Федорова 510. Задачи для ОГЭ с ответами и решениями Линейные уравнения перейти к содержанию задачника Решите уравнение.

Олимпиада по математике 7 класс, задания, уравнения, задачи с ответами

Усвоить школьную программу по математике могут только те, кто проявляет достаточно упорства. На уроках 7 классе учащиеся знакомятся с такими разделами, как степень с натуральным показателем, одночлен и многочлен, линейная функция, системы линейных уравнений с двумя переменными.

Принимая участие в олимпиадах, ученики углубляют свои знания и совершенствуют навыки, приобретенные на уроках. Но, чтобы добиться высокого результата, нужно долго и усердно готовиться.

На нашем сайте вы найдете олимпиадные задания по математике с ответами и решениями. Предложенные задания помогут подготовиться к олимпиаде. Мы советуем вам использовать их в качестве тренажера как на уроках, так и в ходе внеклассной самостоятельной подготовки.

Скачайте задания, заполнив форму!

После того как укажете данные, кнопка скачивания станет активной

Уравнения

1. Оба корня уравнения x2 – ax + 2 являются натуральными числами. Чему равно a?

2. Решите в натуральных числах уравнение:
zx + 1 = (z + 1)2

3. Решите уравнение:
12 – (4х – 18) = (36 + 5х) + (28 – 6х)

4. Найдите решение уравнения:
7x + 3 (x+0,55) = 5,65

5. Решите уравнение:
10у – 13,5 = 2у — 37,5.

6. Преобразуйте в многочлен:
(4х – 5у)2

7. Представьте выражение в виде квадрата двучлена:
4у2 — 12у + 9

8. Решите уравнение:
8у – (3у + 19) = -3(2у — 1)

9. Решите уравнение:
2 – 4х = 0

10. Решите систему уравнений:
{ x+2*y = 12
{ 2*x-3*y = -18

Задачи

Задача №1
Из чисел A, B и C одно положительно, одно отрицательно и одно равно 0. Известно, что A = B (B – C). Какое из чисел положительно, какое отрицательно и какое равно 0? Почему?

Задача №2
Последовательность строится по следующему закону. На первом месте стоит число 7, далее за каждым числом стоит сумма цифр его квадрата, увеличенная на 1. Какое число стоит на 2000 месте?

Задача №3
В XIX-XX веках Россией правили 6 царей династии Романовых. Вот их имена и отчества по алфавиту: Александр Александрович, Александр Николаевич, Александр Павлович, Николай Александрович, Николай Павлович, Павел Петрович. Один раз после брата правил брат, во всех остальных случаях после отца — сын. Как известно, последнего русского царя, погибшего в Екатеринбурге в 1918 году, звали Николаем. Найдите порядок правления этих царей.

Задача №4
Сколько чисел от 1 до 90 делятся на 2, но не делятся на 4?

Задача №5
В трех мешках 114 кг сахара. В первом на 16 кг меньше, чем во втором, а в третьем на 2 кг меньше, чем во втором. Сколько килограммов сахара во втором мешке?

Задача №6
Сколько различных трехзначных чисел можно составить из цифр 1, 2, 3, 4, 5, если цифры в числе не повторяются.

Задача №7
Точка D — середина основания AC равнобедренного треугольника ABC. Точка E — основание перпендикуляра, опущенного из точки D на сторону BC. Отрезки AE и BD пересекаются в точке F. Установите, какой из отрезков BF или BE длиннее.

Задача №8
Пол в гостиной барона Мюнхгаузена вымощен одинаковыми квадратными каменными плитами. Барон утверждает, что его новый ковер (сделанный из одного куска ковролина) закрывает ровно 24 плиты и при этом каждый вертикальный и каждый горизонтальный ряд плит в гостиной содержит ровно 4 плиты, покрытых ковром. Не обманывает ли барон?

Задача №9
Саша выписал первые миллион натуральных чисел, не делящихся на 4. Рома подсчитал сумму 1000 подряд идущих чисел в Сашиной записи. Могло ли у него получиться в результате 20012002?

Задача №10
Автомобиль из A в B ехал со средней скоростью 50 км/ч., а обратно возвращался со скоростью 30 км/ч.. Какова его средняя скорость?

Математические загадки

Загадка №1
Не пользуясь калькулятором и компьютером (в уме) вычислите сумму всех чисел от одного до ста?

Загадка №2
Позавчера Васе было 17 лет. В следующем году ему будет 20 лет. Как такое может быть?

Загадка №3
Два отца и два сына разделили между собой 3 апельсина так, что каждому досталось по одному апельсину. Как это могло получиться?

Загадка №4
На острове живут два племени: молодцы. Которые всегда говорят правду, и лжецы, которые всегда лгут. Путешественник встретил островитянина, спросил его, кто он такой, и когда услышал, что он из племени молодцов, нанял его в проводники. Они пошли и увидели вдали другого островитянина, и путешественник послал своего проводника спросить его, к какому племени он принадлежит. Проводник вернулся и сказал, что тот утверждает, что он из племени молодцов. Спрашивается: был проводник молодцом или лгуном?

Загадка №5
В двух футбольных лигах в сумме 39 команд. Команда играет с каждой командой из своей лиги по одному разу; при этом никаких матчей между лигами не происходит. За победу полагается 3 очка, за ничью — 1 очко, за проигрыш — 0. В прошлом году в одной лиге состоялось на 171 матч больше, чем в другой. Команда «Чемпионы», входящая в одну из лиг, проиграла всего три матча и набрала 32 очка.
Вопрос: со сколькими командами играли «Чемпионы» и сколько раз они сыграли вничью?

Ответы к уравнениям

Уравнение№ 1№ 2№ 3№ 4№ 5
Ответa = 3z = 2
x = 3
x = — 15¹/₃x = 0,4y = -3
Уравнение№ 6№ 7№ 8№ 9№ 10
Ответ16х2 — 40ху + 25у2(2у — 3)2y = 2x = 0
x=0,8
x = 0,6

Ответы к задачам

Задача 1
Если A = 0, то либо B = 0, либо B – C = 0. Ни то, ни другое невозможно. Поэтому A не 0. Если B = 0, то и A = 0. Это тоже невозможно. Поэтому B не 0. Следовательно, C = 0, и равенство из условия задачи можно переписать в виде A = B. Отсюда следует, что B > 0. Значит, B положительно, а A – отрицательно.

Задача 2
Так как 2000 = 3 x 666 + 2, то 2000-м месте стоит число 5.

Задача 3
Павел Петрович, Александр Павлович, Николай Павлович, Александр Николаевич, Александр Александрович, Николай Александрович.

Задача 4
23

Задача 5
44 кг

Задача 6
60 чисел

Задача 7
Отрезок BE длиннее

Задача 8
Примером такой клетчатой фигуры может служить квадрат 6 на 6 без двух подходящих обобщенных диагоналей. Конечно, если трактовать это как ковер в гостиной, получится нечто экстравагантное, но ведь барон не зря слыл незаурядным человеком.

Задача 9
Из любых трёх чисел, идущих в Сашиной записи подряд, одно имеет остаток 1 пр делении на 4, другое – остаток 2, а оставшееся – остаток 3. Значит их сумма при делении на 4 даёт остаток 2. Среди первых 999 Роминых чисел есть ровно 333 таких тройки, сумма чисел в них даёт при делении на 4 такой же остаток, как 333 • 2, то есть 2. Оставшееся число на 4 не делится, поэтому вся сумма не может также давать остаток 2. А 20012002 даёт именно этот остаток.

Задача 10
37,5 км/ч

Ответы на загадки

Загадка 1
5050

Загадка 2
Если нынешний день 1 января, а у Васи день Рождения тридцать первого декабря. Позавчера, т.е. тридцатого декабря ему было еще семнадцать лет. Вчера, т.е. тридцать первого декабря исполнилось восемнадцать лет. В этом году исполнится девятнадцать лет, а в следующем году двадцать лет.

Загадка 3
Всего деливших было трое: дед, его сын и внук

Загадка 4
На острове на данный вопрос никто не мог ответить ничего, кроме того, что он молодец. Так как проводник воспроизвел правильно этот единственно возможный ответ, то ясно, что он молодец.

Загадка 5
«Чемпионы» играли с 23 командами (следовательно, в их лиге 24 команды, а в другой — 15) и сыграли вничью 14 матчей из 23.

Скачайте задания, заполнив форму!

После того как укажете данные, кнопка скачивания станет активной

Другие классы
Обновлено: , автор: Валерия Токарева

Страница не найдена

Новости

10 май

Число погибших в результате серии взрывов около женской школы в Кабуле достигло 85, количество пострадавших превысило 150.

9 май

Эксперты Федерального института педагогических измерений рассказали о самых распространённых ошибках на Едином государственном экзамене (ЕГЭ).

9 май

Афганский телеканал TOLOnews проинформировал о гибели 63 человек в результате серии взрывов около женской школы в Кабуле. Об этом сообщили источники и родственники погибших.

9 май

Число погибших в результате серии взрывов около женской школы в Кабуле превысило 50, пострадавших более 100, сообщает TOLOnews со ссылкой на данные Министерства внутренних дел.

9 май

В колледжах и школах России началась проверка программ обучения истории на предмет полноты сведений о Великой Отечественной войне, заявил РИА Новости министр просвещения Сергей Кравцов.

8 май

По меньшей мере 15 человек погибли и 20 получили ранения в результате серии взрывов в Кабуле, в частности, атака произошла около школы, когда ученики покидали здание, пишет TOLOnews со ссылкой на источник в МВД.

8 май

По меньшей мере 15 человек погибли и 20 получили ранения в результате серии взрывов в Кабуле, в частности, атака произошла около школы, когда ученики покидали здание, пишет Tolo News со ссылкой на источник в МВД.

Решение уравнений с дробями — как решать дробные уравнения

Понятие дроби

Прежде чем отвечать на вопрос, как найти десятичную дробь, разберемся в основных определениях, видах дробей и разницей между ними.

Дробь — это запись числа в математике, в которой a и b — числа или выражения. По сути, это всего лишь одна из форм, в которое можно представить число. Есть два формата записи:

        
  • обыкновенный вид — ½ или a/b,
  •     
  • десятичный вид — 0,5.

Над чертой принято писать делимое (число, которое делим) — числитель. А под чертой всегда находится делитель (на сколько делим), его называют знаменателем. Черта между числителем и знаменателем означает деление.

Дроби бывают двух видов:

        
  1. Числовые — состоят из чисел. Например, 2/7 или (1,8 — 0,3)/5.
  2.     
  3. Алгебраические — состоят из переменных. Например, (x + y)/(x — y). Значение дроби зависит от данных значений букв.

Дробь называют правильной, когда ее числитель меньше знаменателя. Например, 4/9 и 23/57.

Неправильная дробь — та, у которой числитель больше знаменателя или равен ему. Например, 13/5. Такое число называют смешанным — читается так: «две целых три пятых», а записывается — 2 3\5.

                                                                       
Основные свойства дробей
            
                    
  1. Дробь не имеет значения, при условии, если делитель равен нулю.
  2.                 
  3. Дробь равна нулю, если числитель равен нулю, а знаменатель — нет.
  4.                 
  5. Две дроби a/b и c/d называются равными, если a * d = b * c.
  6.                 
  7. Если числитель и знаменатель умножить или разделить на одно и то же натуральное число, то получится равная ей дробь
  8.             
            

Действия с дробями можно выполнять те же, что и с обычными числами: складывать, вычитать, умножать и делить. Также, дроби можно сравнивать между собой и возводить в степень.

Понятие уравнения

Уравнение — это математическое равенство, в котором неизвестна одна или несколько величин. Наша задача — найти неизвестные числа так, чтобы при их подстановке в пример получилось верное числовое равенство. Давайте на примере:

        
  • Возьмем выражение 4 + 5 = 9. Это верное равенство, потому что 4+5 действительно 9. Если бы вместо 9 стояло любое другое число — мы бы сказали, что числовое равенство неверное.
  •     
  • Уравнением можно назвать выражение 4 + x = 9, с неизвестной переменной x, значение которой нужно найти. Результат должен быть таким, чтобы знак равенства был оправдан, и левая часть равнялась правой.

Корень уравнения — то самое число, которое уравнивает выражения справа и слева, когда мы подставляем его на место неизвестной. В таком случае афоризм «зри в корень» — очень кстати при усердном решении уравнений.

Равносильные уравнения — это те, в которых совпадают множества решений. Другими словами, у них одни и те же корни.

Решить уравнение значит найти все его корни или убедиться, что корней нет.

Алгебраические уравнения могут быть разными, самые часто встречающиеся — линейные и квадратные. Расскажем и про них.

                                                                                                 
Линейное уравнение выглядит таках + b = 0, где a и b — действительные числа.             

Что поможет в решении:

            
                    
  • если а не равно нулю, то у уравнения единственный корень: х = -b : а;
  •                 
  • если а равно нулю — у уравнения нет корней;
  •                 
  • если а и b равны нулю, то корень уравнения — любое число.
  •             
            
Квадратное уравнение выглядит так:ax2 + bx + c = 0, где коэффициенты a, b и c — произвольные числа, a ≠ 0.

Приходите решать увлекательные задачки по математике в детскую школу Skysmart. Поможем разобраться в сложной теме, подтянем оценки и покажем, что математика может быть захватывающим приключением.

Запишите ребенка на бесплатный вводный урок: познакомим с форматом, выявим пробелы и наметим индивидуальную программу обучения.

Ты можешь записаться на онлайн-уроки по математике для учеников 1-11 классов!

Понятие дробного уравнения

Дробное уравнение — это уравнение с дробями. Да, вот так просто. Но это еще не все. Чаще всего неизвестная стоит в знаменателе. Например, вот так:

Такие уравнения еще называют дробно-рациональными. В них всегда есть хотя бы одна дробь с переменной в знаменателе.

Если вы видите в знаменателях числа, то это уравнения либо линейные, либо квадратные. Решать все равно нужно, поэтому идем дальше. Примеры:

На алгебре в 8 классе можно встретить такое понятие, как область допустимых значений — это множество значений переменной, при которых это уравнение имеет смысл. Его используют, чтобы проверить корни и убедиться, что решение правильное.

Мы уже знаем все важные термины, их определения и наконец подошли к самому главному — сейчас узнаем как решить дробное уравнение.

 

Как решать уравнения с дробями

                                                                       
Универсальный алгоритм решения
            
                    
  1. Определить область допустимых значений.
  2.                 
  3. Найти общий знаменатель.
  4.                 
  5. Умножить каждый член уравнения на общий знаменатель и сократить полученные дроби. Знаменатели при этом пропадут.
  6.                 
  7. Раскрыть скобки, если нужно и привести подобные слагаемые.
  8.                 
  9. Решить полученное уравнение.
  10.                 
  11. Сравнить полученные корни с областью допустимых значений.
  12.                 
  13. Записать ответ, который прошел проверку.
  14.             
            

А теперь еще несколько способов, которые пригодятся ребенку на уроках математики.

1. Метод пропорции

Чтобы решить уравнение методом пропорции, нужно привести дроби к общему знаменателю. А само правило звучит так: произведение крайних членов пропорции равно произведению средних. Проверим, как это работает.

Итак, у нас есть линейное уравнение с дробями:

В левой части стоит одна дробь — оставим без преобразований. В правой части видим сумму, которую нужно упростить так, чтобы осталась одна дробь.

Как решаем:

После того, как в левой и правой части осталась одна дробь, можно применить метод пропорции и перемножить крест-накрест числители и знаменатели.

2. Метод избавления от дробей

Возьмем то же самое уравнение, но попробуем решить его по-другому.

В уравнении есть две дроби, от которых мы очень хотим избавиться. Вот, как это сделать:

        
  • подобрать число, которое можно разделить на каждый из знаменателей без остатка;
  •     
  • умножить на это число каждый член уравнения.

Ищем самое маленькое число, которое делится на 5 и 9 и без остатка — 45 как раз подходит. Умножаем каждый член уравнения на 45 и избавляемся от знаменателей. Вуаля!

Вот так просто мы получили тот же ответ, что и в прошлый раз.

                                                                       
Что еще важно учитывать при решении
            
                    
  • если значение переменной обращает знаменатель в 0, значит это неверное значение;
  •                 
  • делить и умножать уравнение на 0 нельзя.
  •             
            

А вот и полезные видео для закрепления материала:

Примеры решения дробных уравнений

Чтобы стать успешным в любом деле, нужно чаще практиковаться. Мы уже знаем, как решаются дробные уравнения — давайте перейдем к решению задачек.

Пример 1. Решить дробное уравнение: 1/x + 2 = 5.

Как решаем:

        
  1. Вспомним правило х ≠ 0. Это значит, что область допустимых значений: х — любое число, кроме нуля.
  2.     
  3. Отсчитываем справа налево в числителе дробной части три знака и ставим запятую.
  4.     
  5. Избавимся от знаменателя. Умножим каждый член уравнения на х.     

    1 + 2x = 5х

        
  6.     
  7. Решим обычное уравнение.     

    5x — 2х = 1

        

    3x = 1

        

    х = 1/3

        

Ответ: х = 1/3.

Пример 2. Найти корень уравнения

Как решаем:

        
  1. Область допустимых значений: х ≠ −2.
  2.     
  3. Умножим обе части уравнения на выражение, которое сократит оба знаменателя: 2(х+2)
  4.     
  5. Избавимся от знаменателя. Умножим каждый член уравнения на х.     

        
  6.     
  7. Переведем новый множитель в числитель..     

        
  8.     
  9. Сократим левую часть на (х+2), а правую на 2.     

    4 = х + 2

        

    х = 4 — 2 = 2

        

Ответ: х = 2.

Пример 3. Решить дробное уравнение:

Как решаем:

        
  1. Найти общий знаменатель:     

    3(x-3)(x+3)

        
  2.     
  3. Умножим обе части уравнения на общий знаменатель. Сократим. Получилось:     

    3(x+3)(x+3)+3(x-3)(x-3)=10(x-3)(x+3)+3*36

        
  4.     
  5. Выполним возможные преобразования. Получилось квадратное уравнение:     

    x2-9=0

        
  6.     
  7. Решим полученное квадратное уравнение:     

    x2=9

        
  8.     
  9. Получили два возможных корня:     

    x1=−3, x2=3

        

    х = 4 — 2 = 2

        
  10.     
  11. Если x = −3, то знаменатель равен нулю:     

    3(x-3)(x+3)=0

        

    Если x = 3 — знаменатель тоже равен нулю.

        
  12.     
  13. Вывод: числа −3 и 3 не являются корнями уравнения, значит у данного уравнения нет решения.

Ответ: нет решения.

Если нужно решить уравнение с дробями быстро — поможет онлайн-калькулятор дробей. Пользуйтесь им, если уже разобрались с темой и щелкаете задачки легко и без помощников:

M / J по математике 7 класс — 1205040

Расчет налога и чаевых:

Следите за Хейли и Кенной, когда они оценивают чаевые и налог с продаж в торговом центре, ресторанах и парикмахерских в этом интерактивном руководстве.

Тип: Оригинальное учебное пособие для учащихся

Простой интерес:

Рассчитывайте простые проценты и оценивайте ежемесячные платежи вместе с кредитным специалистом по имени Джордан в этом интерактивном руководстве.

Тип: Оригинальное учебное пособие для учащихся

Налоги, сборы и комиссия:

Изучите налог с продаж, сборы и комиссию, следуя инструкциям представителя службы поддержки по имени Джулиан в этом интерактивном руководстве.

Тип: Оригинальное учебное пособие для учащихся

Углы детской площадки, часть 1:

Изучите дополнительные и дополнительные углы вокруг игровой площадки с Джейкобом в этом интерактивном руководстве.

Это первая часть из серии, состоящей из двух частей. Нажмите, чтобы открыть «Углы детской площадки: Часть 2».

Тип: Оригинальное учебное пособие для учащихся

Углы игровой площадки: Часть 2:

Помогите Джейкобу написать и решить уравнения, чтобы найти недостающие угловые меры, основанные на соотношении между углами, которые в сумме составляют 90 градусов и 180 градусов, в этом интерактивном учебном пособии, посвященном игровой площадке.

Это вторая часть из серии, состоящей из двух частей. Щелкните , чтобы открыть Углы игровой площадки: Часть 1.

Тип: Оригинальное учебное пособие для учащихся

Математические модели и социальное дистанцирование:

Узнайте, как математические модели могут показать, почему социальное дистанцирование во время эпидемии или пандемии важно, в этом интерактивном руководстве.

Тип: Оригинальное учебное пособие для учащихся

Профессор Э. Квал. Часть 1: 2-х ступенчатые уравнения:

Профессор Э. Квал научит вас решать и проверять двухэтапные уравнения в этом интерактивном руководстве.

Это первая часть из двух частей, посвященных решению двухэтапных уравнений.

Тип: Оригинальное учебное пособие для учащихся

Балансировка машины:

Используйте модели для решения задач баланса на космической станции в этом интерактивном учебнике по математике и естествознанию.

Тип: Оригинальное учебное пособие для учащихся

Пицца Пи: Окружность:

Исследуйте происхождение числа Пи как отношения окружности к диаметру круга. В этом интерактивном руководстве вы будете работать с формулой окружности, чтобы определить длину окружности, и работать в обратном направлении, чтобы определить диаметр и радиус окружности.

Тип: Оригинальное учебное пособие для учащихся

Введение в вероятность:

Узнайте, как рассчитать вероятность простых событий, эта вероятность — это вероятность того, что событие произойдет, и что некоторые события могут произойти с большей вероятностью, чем другие, в этом интерактивном руководстве.

Тип: Оригинальное учебное пособие для учащихся

Изучение среднего абсолютного отклонения: крылатка:

Сравните несколько образцов крылатки, чтобы сделать обобщение о популяции, проанализировав средние абсолютные отклонения образцов и их распределение в этом интерактивном руководстве.

Тип: Оригинальное учебное пособие для учащихся

Алиса в стране математики:

Помогите Алисе обнаружить, что составные вероятности можно определить путем вычислений или рисования древовидных диаграмм в этом интерактивном руководстве.

Тип: Оригинальное учебное пособие для учащихся

Пицца Пи: Площадь:

В этом интерактивном руководстве узнайте, как рассчитать площадь кругов в единицах числа «пи» и с помощью аппроксимации числа «пи».Вы также столкнетесь с ситуациями с неправильной площадью, которые потребуют использования формулы площади круга.

Тип: Оригинальное учебное пособие для учащихся

Куда делись все скраб-сойки ?:

Изучите ограничивающие факторы экосистемы Флориды и опишите, как эти ограничивающие факторы влияют на одно коренное население — Флоридскую скраб-сойку — с помощью этого интерактивного учебного пособия.

Тип: Оригинальное учебное пособие для учащихся

Удивительные приключения:

Узнайте, как объяснить значение аддитивной инверсии, определить аддитивную инверсию данного рационального числа и обосновать свой ответ числовой линией в этом оригинальном руководстве.

Тип: Оригинальное учебное пособие для учащихся

По горячим следам:

Изучите, как температура влияет на скорость химических реакций, в этом интерактивном руководстве.

Тип: Оригинальное учебное пособие для учащихся

Да или нет ГМО ?:

Узнайте, что такое генная инженерия и некоторые применения этой технологии.В этом интерактивном руководстве вы получите представление о некоторых преимуществах и потенциальных недостатках генной инженерии. В конечном итоге вы сможете критически относиться к генной инженерии и написать аргумент, описывающий вашу собственную точку зрения на ее влияние.

Тип: Оригинальное учебное пособие для учащихся

Плавание кругами:

В этом интерактивном руководстве научитесь решать задачи, касающиеся окружности и площади круга бассейнов.

Тип: Оригинальное учебное пособие для учащихся

Масштаб округляется в большую сторону:

В этом интерактивном руководстве научитесь использовать архитектурные масштабные чертежи для строительства новой арены для лошадей и решать проблемы, связанные с масштабными чертежами. К концу вы сможете рассчитать фактическую длину, используя масштаб и пропорции.

Тип: Оригинальное учебное пособие для учащихся

Спорящий Марс:

Узнайте, как определять явные доказательства и понимать неявное значение текста.

В этом руководстве вы научитесь определять аргументы или утверждения говорящего.Вы также узнаете, как оценивать доказательства и аргументы, представленные в речи.

Тип: Оригинальное учебное пособие для учащихся

Улыбки:

В этом интерактивном задании по решению задач учащиеся применяют алгебраические рассуждения для определения «стоимости» отдельных типов лиц по сумме хмурых взглядов, улыбок и нейтральных лиц.На этой странице представлены три графические задачи, связанные с решением систем уравнений, а также советы по обдумыванию проблемы, решения и других подобных проблем.

Тип: Задача по решению проблем

Титаник 1:

Это задание просит студентов вычислить вероятности, используя информацию, представленную в двухсторонней таблице частот.

Тип: Задача по решению проблем

Анна в Вашингтоне:

Цель этого задания — дать учащимся возможность решить сложную многоступенчатую процентную задачу, к которой можно подойти разными способами.Студентам предлагается найти стоимость еды без учета налогов и чаевых, если указана общая стоимость еды. Задача может иллюстрировать несколько стандартов в зависимости от предварительных знаний учащихся и подхода, используемого для решения проблемы.

Тип: Задача по решению проблем

Книги со скидкой:

Цель этого задания — помочь учащимся увидеть два разных способа взглянуть на проценты как на уменьшение, так и на увеличение исходной суммы.Кроме того, ученики должны превратить словесное описание нескольких операций в математические символы. Это требует преобразования простых процентов в десятичные числа, а также определения эквивалентных выражений без переменных.

Тип: Задача по решению проблем

Эквивалентные выражения ?:

Учащимся предлагается определить, эквивалентны ли два выражения, и объяснить свои рассуждения.

Тип: Задача по решению проблем

Рыболовные приключения 2:

Студентам предлагается написать и решить неравенство, чтобы определить количество людей, которые могут безопасно арендовать лодку.

Тип: Задача по решению проблем

Угадай мой номер:

В этой задаче учащимся предлагается представить последовательность операций с помощью выражения, а затем написать и решить простые уравнения.Задача представлена ​​в виде игры и позволяет учащимся визуализировать математические операции. Было бы разумно сначала сыграть в похожую игру парами, а затем попросить учеников записать операции, чтобы вычислить числа друг друга.

Тип: Задача по решению проблем

Мили в Километры:

В этом задании учащихся просят написать два выражения из словесных описаний и определить, эквивалентны ли они.В выражениях используются как проценты, так и дроби. Это задание лучше всего подходит для обсуждения в классе, поскольку в формулировке проблемы имеется некоторая двусмысленность.

Тип: Задача по решению проблем

Усадка:

Учащимся предлагается определить изменение роста в дюймах при постоянной скорости изменения в сантиметрах.Ответ округляется до ближайших полдюйма.

Тип: Задача по решению проблем

Комплект спортивного инвентаря:

Учащегося просят написать и решить неравенство в соответствии с контекстом.

Тип: Задача по решению проблем

Восемь кругов:

Учащимся предлагается найти площадь затененной области, используя диаграмму и предоставленную информацию.Цель этого задания — улучшить понимание учащимися местности.

Тип: Задача по решению проблем

Поэтажный план:

Целью этого задания является перевод учащихся между измерениями, приведенными на чертеже в масштабе, и соответствующими измерениями объекта, представленными на чертеже в масштабе.Если бы они использовались в учебных заведениях, было бы хорошо, если бы учащиеся имели возможность увидеть другие методы решения, возможно, попросив учащихся с разными подходами объяснить свои стратегии классу. Студентам, которые могут решить эту проблему, только предварительно преобразовав линейные измерения, будет сложно решить задачи, в которых указаны только измерения площади.

Тип: Задача по решению проблем

Расстояния на числовой прямой 2:

Цель этого задания состоит в том, чтобы укрепить понимание учащимися рациональных чисел как точек на числовой прямой и предоставить им наглядный способ понимания того, что сумма числа и его аддитивная обратная величина (обычно называемая его «противоположностью») равно нулю.

Тип: Задача по решению проблем

Сравнение точек замерзания:

В этом задании учащиеся отвечают на вопрос о разнице между двумя отрицательными числами.

Тип: Задача по решению проблем

Купон или скидка:

В этом задании ученикам предлагается реальная проблема, связанная с ценой продаваемого товара.Чтобы ответить на вопрос, учащиеся должны представить проблему, указав переменную и связанные с ней величины, а затем написать и решить уравнение.

Тип: Задача по решению проблем

Операции на числовой линии:

Цель этого задания — помочь учащимся укрепить понимание чисел со знаком как точек на числовой прямой и понять геометрическую интерпретацию сложения и вычитания чисел со знаком.В Стандартах Флориды есть тонкое различие между дробью и рациональным числом. Дроби всегда положительны, и если рассматривать символ ab как дробь, можно интерпретировать его как части равного размера, где b частей составляют одно целое.

Тип: Задача по решению проблем

Повторяющееся десятичное число как приближение:

Учащегося просят выполнить длинное деление, в результате чего получается повторяющаяся десятичная дробь, а затем использовать умножение, чтобы «проверить» свой ответ.Цель задания — заставить учащихся задуматься о значении повторения десятичного представления через приближение.

Тип: Задача по решению проблем

Разделение призовых денег:

Учащимся предлагается определить, как распределить денежные призы между тремя классами в зависимости от вклада каждого класса.

Тип: Задача по решению проблем

Набор песка под качелями:

Ученики 7-го класса средней школы Санвью помогали отремонтировать игровую площадку для детсадовцев в соседней начальной школе. Городские правила требуют, чтобы песок под качелями был не менее 15 дюймов в глубину.Когда они стартовали, песок под обоими качелями был всего 12 дюймов в глубину. Прямоугольная область под маленькими качелями имеет размеры 9 футов на 12 футов, и для увеличения глубины на 3 дюйма потребовалось 40 мешков с песком. Сколько мешков с песком понадобится ученикам, чтобы покрыть прямоугольную область под большими качелями, если она в 1,5 раза длиннее и в 1,5 раза шире, чем область под малыми качелями?

Тип: Задача по решению проблем

Арт-класс, вариация 1:

Студентам предлагается использовать соотношения и пропорциональные рассуждения для сравнения смесей красок численно и графически.

Тип: Задача по решению проблем

Шахматный клуб:

Эта задача включает процентное увеличение одной части с процентным уменьшением оставшейся части и просит учащихся найти общее процентное изменение. Проблема может быть решена с использованием пропорций, расчетов или написания набора уравнений.

Тип: Задача по решению проблем

Сравнение лет:

Студентам предлагается сравнить египетский, григорианский и юлианский методы измерения года.

Тип: Задача по решению проблем

Готовим из целой чашки:

Студентам предлагается использовать пропорциональные рассуждения, чтобы ответить на ряд вопросов в контексте рецепта.

Тип: Задача по решению проблем

Такси Готэма:

Цель этого задания — дать учащимся возможность решить задачу, состоящую из нескольких шагов, к которой можно подойти разными способами. Это можно сделать, составив таблицу, которая помогает проиллюстрировать структуру ставок такси для разных пройденных расстояний и с небольшой настойчивостью приводит к решению, в котором используется арифметика.Также можно рассчитать единицу скорости (в долларах за милю) и использовать ее, чтобы найти расстояние напрямую, не составляя таблицы.

Тип: Задача по решению проблем

Обнаружение увеличения на 10%:

За первую неделю книжную ярмарку посетили 5000 человек.За вторую неделю количество посетителей увеличилось на 10%. Сколько человек посетило книжную ярмарку за вторую неделю?

Тип: Задача по решению проблем

Бег Молли:

В этом задании учащимся предлагается решить задачу в контексте постоянной скорости.Эта задача обеспечивает переход от работы с отношениями, содержащими целые числа, к отношениям, содержащим дроби. Эту проблему можно рассматривать по-разному; в частности, эта задача также дает возможность студентам работать с интерпретацией деления «Сколько в одной группе?».

Тип: Задача по решению проблем

Музыкальные компании, Вариант 1:

Эта задача требует сравнения ставок, где одна дается в единицах, а другая — нет.См. «Музыкальные компании. Вариант 2», где описана задача с очень похожей настройкой, но она намного сложнее и поэтому иллюстрирует.

Тип: Задача по решению проблем

Музыкальные компании, Вариант 2:

Эта проблема состоит из нескольких этапов.Для решения проблемы необходимо вычислить: стоимость акций TunesTown; общая стоимость предложения BeatStreet о 20 миллионах акций по 25 долларов за акцию; разница между этими двумя суммами; а стоимость каждой из дополнительных 2 миллионов акций, предлагаемых MusicMind, равна разнице.

Тип: Задача по решению проблем

Гонки роботов:

Учащиеся должны использовать предоставленную информацию, чтобы ответить на вопросы о гонках роботов.

Тип: Задача по решению проблем

Распродажа!:

Студентам предлагается определить, какой вариант продажи приводит к наибольшему процентному снижению стоимости.

Тип: Задача по решению проблем

Продажа компьютеров:

В прошлом месяце отдел продаж магазина электроники продал 48 компьютеров.Менеджер магазина хочет побудить отдел продаж продавать больше компьютеров и собирается дать всем членам отдела продаж бонус, если в следующем месяце количество проданных компьютеров увеличится на 30%. Сколько компьютеров должно продать отдел продаж, чтобы получить бонус? Объясните свои рассуждения.

Тип: Задача по решению проблем

Биржевые свопы, вариант 2:

Студентов просят решить задачу, используя пропорциональные рассуждения в реальном контексте, чтобы определить количество акций, необходимых для совершения покупки акций.

Тип: Задача по решению проблем

Биржевые свопы, вариант 3:

Студентов просят решить задачу с соотношением шагов в реальном мире.

Тип: Задача по решению проблем

Налог и чаевые:

Пообедав в любимом ресторане, вы знаете, что счет до налогообложения составляет 52 доллара.60 и что ставка налога с продаж составляет 8%. Вы решаете оставить официанту чаевые в размере 20% от суммы до налогообложения. Сколько оставить официанту? Сколько будет стоить общий счет, включая налоги и чаевые?

Тип: Задача по решению проблем

Цена хлеба:

Цель этого задания — вычислить процент увеличения и относительную стоимость в реальном контексте.Инфляция, одна из главных идей в экономике, — это рост цен на товары и услуги с течением времени. Это зависит от имеющейся у вас суммы денег.

Тип: Задача по решению проблем

Тренировочная практика:

В этом упражнении студенту предлагается использовать единичную ставку и пропорциональное рассуждение, чтобы определить, кто из двух бегунов самый быстрый.

Тип: Задача по решению проблем

Две школы танца:

Цель этого задания — увидеть, насколько хорошо учащиеся понимают коэффициенты и рассуждают с ними.

Тип: Задача по решению проблем

Мистер.Классу Бригга нравится математика:

По результатам опроса, проведенного на уроке математики мистера Бриггса, 67% студентов заявили, что математика является их любимым учебным предметом. В классе находится редактор школьной газеты, и он хочет написать для газеты статью о том, что математика — самый популярный предмет в школе. Объясните, почему это неверный вывод, и предложите способ собрать более точные данные, чтобы определить, какой предмет является наиболее популярным.

Тип: Задача по решению проблем

Линейщики наступления:

В этом задании учащиеся могут предположить о различиях и сходстве в двух группах с чисто визуальной точки зрения, а затем подкрепить свои сравнения соответствующими мерами центра и изменчивости.Это подтвердит, что многое можно почерпнуть просто из визуального сравнения соответствующих графиков, особенно аналогичных по масштабу.

Тип: Задача по решению проблем

Подбрасывание цилиндров:

Цель этого задания — предоставить студентам возможность определить экспериментальные вероятности путем сбора данных.Цилиндрические объекты, используемые в этой задаче, обычно имеют три разных положения покоя, но не все из них могут быть одинаково вероятными, а некоторые могут быть крайне маловероятными или невозможными при подбрасывании объекта. Более того, получение вероятностей результатов возможно только за счет использования долгосрочных относительных частот. Это связано с тем, что эти цилиндры не обладают симметрией тех же типов, что и объекты, которые часто используются в качестве игральных костей, такие как кубы или тетраэдры, где каждый результат одинаково вероятен.

Тип: Задача по решению проблем

Сколько кнопок ?:

Этот ресурс включает в себя простую деятельность по сбору данных, которая предоставляет данные, которые учащиеся объединяют в таблицу. Затем их просят обратиться к данным и определить вероятность различных результатов.

Тип: Задача по решению проблем

Избирательный опрос, Вариант 2:

Эта задача знакомит с фундаментальными статистическими идеями использования сводных данных (статистики) из случайных выборок для вывода (обоснованных выводов) о характеристиках (параметрах) совокупности.В задаче, построенной на сценарии избирательного опроса, совокупность — это весь седьмой класс, интересующая неизвестная характеристика (параметр) — это доля членов класса, голосующих за конкретного кандидата, а итоговая сводка (статистика) — это наблюдаемые доля избирателей, отдавших предпочтение кандидату в случайной выборке учащихся. Вариант 2 проводит студентов через физическое моделирование для создания пропорций выборки путем выборки и повторной выборки шариков из коробки.

Тип: Задача по решению проблем

Избирательный опрос, Вариант 1:

Эта задача знакомит с фундаментальными статистическими идеями использования сводных данных (статистики) из случайных выборок для вывода (обоснованных выводов) о характеристиках (параметрах) совокупности.Эта задача преследует две важные цели: увидеть необходимость случайной выборки и использовать рандомизацию для исследования поведения выборочной статистики. Они вводят основные идеи статистического вывода и могут быть выполнены с минимальными знаниями вероятности.

Тип: Задача по решению проблем

Время ожидания:

По мере развития стандартов статистики и вероятности учащиеся еще не будут знать правила вероятности для сложных событий.Таким образом, моделирование используется для поиска приблизительного ответа на эти вопросы. Фактически, часть b будет вызовом для студентов, которые действительно знают правила вероятности, еще больше демонстрируя возможности моделирования в предоставлении относительно простых приблизительных ответов на самые разные проблемы.

Тип: Задача по решению проблем

Катящаяся игральная кость:

Это задание предназначено для использования в классе.Учащиеся объединяют результаты многих повторений случайного явления (бросания кубиков) и сравнивают свои результаты с теоретическим ожиданием, которое они получают, рассматривая все возможные результаты броска двух кубиков. Это дает им конкретный пример того, что мы подразумеваем под долгосрочной относительной частотой.

Тип: Задача по решению проблем

Прокатка дважды:

Цель этого задания — вычислить теоретическую вероятность сложного события.Учителя могут пожелать подчеркнуть различие между теоретической и экспериментальной вероятностями для этой проблемы. Для студентов, которые учатся различать теоретическую и экспериментальную вероятность, было бы хорошо найти экспериментальную вероятность либо до, либо после того, как студенты вычислили теоретическую вероятность.

Тип: Задача по решению проблем

Сидя друг напротив друга:

Цель этого задания — вычислить теоретическую вероятность конфигурации рассадки.В этой задаче есть 24 возможных конфигурации четырех друзей за столом. Учащиеся могут нарисовать все 24 конфигурации для решения задачи, но это занимает много времени, и поэтому их следует поощрять искать более систематический метод.

Тип: Задача по решению проблем

Бревенчатая поездка:

Учащимся предлагается решить неравенство, чтобы ответить на реальный вопрос.

Тип: Задача по решению проблем

Проблема с возрастным словом:

Этот учебник показывает студентам, как составить и решить возрастную задачу.В руководстве также показано, как проверить свою работу с помощью подстановки.

Тип: Учебное пособие

Проблема с возрастным словом:

Учащиеся научатся составлять и решать задачу о возрастных словах.

Тип: Учебное пособие

Обнаружение вероятности:

Это видео демонстрирует несколько примеров определения вероятности случайных событий.

Тип: Учебное пособие

Окружность круга:

В этом видео показано, как найти длину окружности, расстояние по окружности, учитывая площадь.

Тип: Учебное пособие

Составные пробелы:

В этом видео рассказывается, как создавать пробные пространства в виде древовидных диаграмм, списков и таблиц.

Тип: Учебное пособие

Вероятность прокатки штампа:

Видео покажет, как использовать таблицу для определения вероятности сложного события.

Тип: Учебное пособие

Площадь круга:

В этом видео вы увидите, как мы находим площадь круга при заданном диаметре.

Тип: Учебное пособие

Проблема со словом пропорции:

Это вводное видео демонстрирует базовые навыки написания и решения основного уравнения для пропорциональной зависимости.

Тип: Учебное пособие

Решение пропорции с неизвестной переменной:

Вот вводное видео, в котором объясняются основные аргументы в пользу решения пропорций и показаны три различных метода решения пропорций, которые вы будете использовать позже для решения более сложных задач.

Тип: Учебное пособие

Настройка пропорций для решения проблем со словами:

В этом вводном видео показаны некоторые основные примеры записи двух соотношений и приравнивания их друг к другу. Это всего лишь шаг 1 при решении задач со словами с пропорциями.

Тип: Учебное пособие

Оценить проблему с дробями:

Посмотрите, как мы решаем задачу определения скорости в метрах в секунду, используя расстояние (в метрах) и время (в секундах).

Тип: Учебное пособие

Процент проблемы со словом:

Узнайте, как найти полную цену, если вы знаете цену со скидкой в ​​этой задаче о процентах.

Тип: Учебное пособие

Добавление отрицательных чисел:

Это видео демонстрирует использование числовой прямой и абсолютного значения для сложения отрицательных чисел.

Тип: Учебное пособие

Умножение целых чисел:

В этом руководстве демонстрируется метод умножения целых чисел на числовую линию.Вы столкнетесь с четырьмя различными комбинациями при умножении целых чисел: (1) положительное, умноженное на положительное, (2) положительное, умноженное на отрицательное, (3) отрицательное, умноженное на отрицательное, (4) отрицательное, умноженное на положительное. Урок доступен в видеоформате, есть викторина для практики.

Тип: Учебное пособие

Решение двухэтапных уравнений:

В этом коротком видео используется как уравнение, так и наглядная модель, чтобы объяснить, почему одни и те же шаги должны использоваться для обеих сторон уравнения при решении для значения переменной.

Тип: Учебное пособие

Преалгебра — дроби и рациональные числа:

Первые дроби, используемые древними цивилизациями, были «единичными дробями». Позже были добавлены другие числители, в результате чего образовались «вульгарные дроби», которые стали нашими современными дробями.Вместе дроби и целые числа образуют «рациональные числа».

Тип: Учебное пособие

Преалгебра — умножение отрицательных чисел:

Когда системы счисления были расширены за счет включения отрицательных чисел, необходимо было сформулировать правила, чтобы умножение было согласованным независимо от знака операндов.

Тип: Учебное пособие

Добавление целых чисел:

Студенты смогут увидеть примеры сложения целых чисел во время просмотра короткого видео и попрактиковаться в сложении целых чисел с помощью онлайн-викторины.

Тип: Учебное пособие

Линейные уравнения с одной переменной:

Этот урок знакомит учащихся с линейными уравнениями с одной переменной, показывает, как их решать, используя свойства равенств сложения, вычитания, умножения и деления, и позволяет учащимся определить, является ли значение решением, существует ли бесконечно много решений или вообще нет решения.Сайт содержит объяснение уравнений и линейных уравнений, как решать уравнения в целом, а также стратегию решения линейных уравнений. Урок также объясняет противоречие (уравнение без решения) и тождество (уравнение с бесконечными решениями). В конце есть пять практических задач, чтобы студенты могли проверить свои знания со ссылками на ответы и объяснениями, как эти ответы были найдены. Также указаны дополнительные ресурсы.

Тип: Учебное пособие

Использование метода пропорций для решения процентных задач:

Этот сайт подробно описывает шаги по использованию метода пропорций для решения трех различных типов процентных задач.Он также включает примеры задач для практического определения части, целого или процентного содержания.

Тип: Учебное пособие

Основные аддитивные цвета:

Этот ресурс помогает пользователю изучить три основных цвета, которые имеют фундаментальное значение для человеческого зрения, изучить различные цвета в видимом спектре, наблюдать получающиеся цвета при добавлении двух цветов и узнать, что такое белый свет.Комбинация текста и виртуального манипулятора позволяет пользователю исследовать эти концепции множеством способов.

Тип: Учебное пособие

Основные субтрактивные цвета:

Пользователь изучит три основных субтрактивных цвета в видимом спектре, исследует результирующие цвета, когда два субтрактивных цвета взаимодействуют друг с другом, и изучит формирование черного цвета.

Тип: Учебное пособие

Преобразование единиц скорости:

На этом уроке учащиеся будут просматривать видеоролик Khan Academy, в котором будет показано, как преобразовывать коэффициенты с использованием единиц скорости.

Тип: Учебное пособие

Умножение дробей:

Видео описывает, как умножать дроби и формулировать ответ в минимальных числах.

Тип: Учебное пособие

Спиннер:

В этом упражнении учащиеся регулируют количество секций на вертушке, а затем проводят имитационные испытания на этом вертушке, чтобы развить концепции вероятности.В таблице рядом с прядильщиком отображается теоретическая вероятность для каждого цветного участка прядильщика и записывается экспериментальная вероятность из испытаний прядения. Это упражнение позволяет студентам исследовать темы экспериментальной и теоретической вероятности, видя их рядом с созданным ими прядильщиком. Это упражнение включает в себя дополнительные материалы, в том числе справочную информацию по затронутым темам, описание того, как использовать приложение, и вопросы исследования для использования с java-апплетом.

Тип: виртуальный манипулятор

Флаер поперечного сечения — Shodor:

С помощью этого онлайн-апплета Java учащиеся используют ползунки для перемещения поперечного сечения конуса, цилиндра, призмы или пирамиды. Это задание позволяет учащимся изучить конические сечения и трехмерные формы, из которых они получены.Это упражнение включает в себя дополнительные материалы, в том числе справочную информацию по затронутым темам, описание того, как использовать приложение, и вопросы исследования для использования с java-апплетом.

Тип: виртуальный манипулятор

Инструмент Круг:

Этот апплет позволяет студентам исследовать отношения между площадью и длиной окружности круга, его радиусом и диаметром.На сайте три раздела: Введение, Исследование и Проблемы.

  • Во вводном разделе учащиеся могут изменять размер круга и видеть, как это влияет на радиус, диаметр и длину окружности. Учащиеся также могут воспроизвести видеоклип, чтобы наглядно увидеть, как связаны эти измерения.
  • Раздел «Исследование» позволяет учащимся собирать точки данных, перетаскивая радиус круга на разную длину, и записывать в таблицу данные для радиуса, диаметра, окружности и площади.Нажатие кнопки x / y позволяет учащимся изучить взаимосвязь между любыми двумя показателями. При нажатии на кнопку графика студенты переходят к графику данных. Они могут нанести любой из четырех показателей на ось x против любого из четырех показателей на оси y.
  • Раздел «Задачи» содержит вопросы, которые ученики должны решить, и записать свои ответы в правильном разделе.

(Подсветка NCTM)

Тип: виртуальный манипулятор

Линейная функциональная машина:

В этом упражнении учащиеся подставляют значения в независимую переменную, чтобы увидеть, каковы выходные данные для этой функции.Затем на основе этой информации они должны определить коэффициент (наклон) и константу (пересечение оси y) для линейной функции. Это упражнение позволяет студентам изучить линейные функции и то, какие входные значения полезны при определении правила линейной функции. Это упражнение включает в себя дополнительные материалы, в том числе справочную информацию по затронутым темам, описание того, как использовать приложение, и вопросы исследования для использования с апплетом Java.

Тип: виртуальный манипулятор

Смеси:

В этом онлайн-упражнении учащиеся применяют свое понимание пропорциональных отношений, добавляя кружки, цветные или нет, к двум разным стопкам, а затем объединяют стопки, чтобы получить требуемый процент цветных кругов.Студенты могут играть в четырех режимах: исследование, неизвестная часть, неизвестное целое или неизвестный процент. Это упражнение также включает в себя дополнительные материалы на вкладках над апплетом, включая справочную информацию о затронутых темах, описание того, как использовать приложение, и вопросы исследования для использования с Java-апплетом.

Тип: виртуальный манипулятор

Графические линии:

Позволяет учащимся получить доступ к декартовой системе координат, в которой можно построить график линейных уравнений и наблюдать за деталями линии и наклона.

Тип: виртуальный манипулятор

Коробчатая диаграмма:

В этом упражнении учащиеся используют предустановленные данные или вводят свои собственные данные, которые будут представлены в виде прямоугольной диаграммы. Это упражнение позволяет учащимся изучать как отдельные, так и расположенные бок о бок диаграммы различных данных.Это упражнение включает в себя дополнительные материалы, в том числе справочную информацию по затронутым темам, описание того, как использовать приложение, и вопросы исследования для использования с апплетом Java.

Тип: виртуальный манипулятор

Интерактивные шарики:

Это интерактивное средство для манипуляций позволяет учащемуся имитировать размещение шариков в сумке и определение вероятности вытаскивания определенных комбинаций шариков.Это позволяет исследовать вероятности нескольких событий, а также вероятность с заменой и без нее. Вкладки над апплетом предоставляют доступ к дополнительным материалам, включая справочную информацию по затронутым темам, описание того, как использовать приложение, и вопросы исследования для использования с апплетом Java.

Тип: виртуальный манипулятор

Advanced Data Grapher:

Это онлайн-утилита для построения графиков, которую можно использовать для создания коробчатых диаграмм, пузырьковых диаграмм, диаграмм рассеяния, гистограмм и диаграмм типа «стержень-лист».

Тип: виртуальный манипулятор

Вероятность Плинко:

Студенты сыграют в классическую игру из популярного шоу. Благодаря этому они могут исследовать вероятность того, что мяч приземлится на каждом из чисел, и обнаружить более точные результаты, полученные при повторном тестировании.Моделирование можно настроить, чтобы повлиять на справедливость и случайность результатов.

Тип: виртуальный манипулятор

Подгонка кривой:

С помощью мыши учащиеся будут перетаскивать точки данных (с их планками ошибок) и мгновенно наблюдать за формой наиболее подходящей полиномиальной кривой.Студенты могут выбрать тип соответствия: линейный, квадратичный, кубический или квартичный. Может отображаться наилучшая или регулируемая посадка.

Тип: виртуальный манипулятор

Ящичный плоттер:

Пользователи выбирают набор данных или вводят свои собственные данные для создания прямоугольной диаграммы.

Тип: виртуальный манипулятор

Инструмент случайного рисования — отдельные испытания (вероятностное моделирование):

Этот виртуальный манипулятор позволяет создать случайную коробку для рисования, выложив до 21 билета с числами от 0 до 11.После выбора билетов, которые нужно положить в коробку, апплет будет выбирать билеты случайным образом. Также есть опция, которая покажет теоретическую вероятность для каждого билета.

Тип: виртуальный манипулятор

Масштаб:

Изучите влияние на периметр и площадь двух прямоугольных форм при изменении масштабного коэффициента.

Тип: виртуальный манипулятор

Common Core: справка по математике для 7-го класса

Учащиеся, нуждающиеся в помощи Common Core: 7th Grade Math, получат большую пользу от нашей интерактивной программы. Мы разбиваем все ключевые элементы, чтобы вы могли получить адекватную справку по Common Core: 7th Grade Math. С необходимыми концепциями обучения и соответствующими практическими вопросами прямо у вас под рукой, вы получите много помощи Common Core: 7th Grade Math в кратчайшие сроки.Получите помощь сегодня с нашей обширной коллекцией важной информации Common Core: 7th Grade Math.

Common Core 7th Grade Math может показаться некоторым родителям пугающим, поскольку Common Core — относительно новое дополнение к школьной программе. Инструменты обучения Varsity Tutors создали ресурс Common Core 7th Grade Math Learn by Concept, чтобы предоставить вашему ребенку дополнительную помощь, в которой он нуждается. Learn by Concept — это бесплатный онлайн-ресурс по общей основной математике для 7-х классов, оформленный в виде интерактивной программы.Имея под рукой разнообразные бесплатные учебные пособия по общей основной математике в 7-м классе, вы можете быть уверены, что у него есть доступ к необходимой помощи.

Общий курс математики для 7-х классов по концепциям разделен на пять основных категорий, включая выражения и уравнения, геометрию, соотношения и пропорции, статистику и вероятность, а также систему счисления. Ваш ребенок может ожидать, что он найдет вопросы о решении уравнений, алгебраических неравенствах, масштабах, окружности, соотношениях, выводе, вероятности, сложении и вычитании с отрицательными числами и многом другом.Вы можете чувствовать себя уверенно, зная, что ваш ребенок имеет доступ ко всей необходимой информации. Каждый вопрос, задаваемый в программе Common Core 7-го класса по математике, напрямую связан с информацией, которую они изучают или будут изучать в 7-м классе.

Как только ваш ребенок выберет тему, в которой он хочет проверить свои способности, он найдет один или несколько вопросов по выбранной им концепции. Каждый примерный вопрос Common Core 7th Grade Math представлен с несколькими вариантами ответов на выбор. Ваш ребенок может уделить столько времени, сколько ему нужно, чтобы ответить на каждый вопрос; нет ограничения по времени.Как только они почувствуют себя комфортно с выбранным ответом, они могут прокрутить вниз и определить, правильно ли они его поняли. Каждый правильный ответ четко обозначен, и за каждым ответом следует подробное объяснение, поэтому вы можете чувствовать себя непринужденно, зная, что вашему ребенку показывают правильный подход к каждой проблеме. Эти бесплатные практические вопросы по общей основной математике для 7-го класса созданы, чтобы дать вашему ученику доступ к наиболее точным и актуальным доступным материалам.

Common Core 7th Grade Math Learn by Concept — не единственный доступный ресурс, который может оказать вашему ребенку необходимую помощь.Вы и ваш ребенок также найдете «Вопрос дня», карточки, практические тесты по конкретным концепциям и полные практические тесты. «Вопрос дня» каждый день предлагает вашему ребенку оригинальный вопрос, который будет держать его разум вовлеченным в предмет. Полностью настраиваемые карточки Common Core по математике для 7-го класса позволяют вашему ребенку создавать концептуальные упражнения с учетом их потребностей. Общие базовые практические тесты по математике для 7-го класса дают вашему ребенку представление о концепциях, с которыми он может столкнуться на реальных тестах, а полные практические тесты могут помочь вам в составлении плана обучения по общей основной математике 7-го класса для вашего ребенка.Все учебные инструменты для репетиторов университетской школы на 100% бесплатны, так что вы можете побудить своего ребенка воспользоваться полным спектром бесплатных ресурсов для обзора общих основ математики для 7-го класса.

Важные математические навыки для семиклассников

Хотите помочь своему семикласснику освоить математику? Вот некоторые навыки, которые ваш ребенок будет изучать в седьмом классе.

Коэффициенты и удельные ставки

Реальные проблемы

Решение реальных проблем, касающихся ставок, соотношений, пропорций и процентов, включая скидки, наценки, уценки, проценты, налоги, чаевые, комиссионные, процентное увеличение или уменьшение.

Пример:

На распродаже со скидкой 25% Марисса покупает юбку за 40,50 долларов. Какова была первоначальная цена юбки? Добавив налог с продаж в размере 6%, какова общая стоимость юбки?

Рецепт требует 3⁄4 стакана сливок на каждые 2 стакана молока. Если количество молока увеличить до 8 стаканов, сколько потребуется стаканов сливок?

Единица скорости изменения

Под переменными понимаются символы для чисел или значений, которые еще не известны — например, x и y — это переменные в y = 2x + 6 .Используя уравнения, таблицы, графики и описания, определите скорость изменения единицы — коэффициент, сравнивающий изменение одной величины с изменением на 1 единицу другой величины.

Пример:

Энтони читает 36 страниц за час. В следующий час он читает 42 страницы. Какова скорость изменения количества страниц, которые он может прочитать за час? Объясните свои рассуждения.

Расчет удельных ставок

Рассчитайте удельные расценки, связанные с отношениями долей, включая отношения длин и площадей, а также количества, измеренные в различных единицах.

Пример:

  • Если человек проходит 1⁄2 мили за каждые часа, какова (единица) скорость, с которой он идет, выраженная в милях в (1) час?
  • Для заполнения 1⁄9 емкости для рыбы требуется 1⁄8 литра воды. Сколько литров воды нужно для заполнения бака?

Связанные

Сложение, вычитание, умножение и деление

Многоступенчатые задачи реального мира

Сложение, вычитание, умножение и деление с положительными и отрицательными рациональными числами в любой форме, включая целые числа, дроби, или десятичные дроби.Поймите, что числа нельзя делить на 0. Используйте эти навыки для решения многоступенчатых реальных задач.

Пример:

  • Если няня получает 13 долларов в час и получает надбавку на 15%, какова будет ее новая почасовая оплата? Что теперь она будет делать за 5 с половиной часов присмотра за детьми? Объясните или проиллюстрируйте свои рассуждения.
  • В Галифаксе низкие температуры (по Фаренгейту) в течение семи дней января были: -12 °, -3 °, 6 °, -14 °, -8 °, 9 °, -1 °. Какая была средняя температура на той неделе? Объясните или проиллюстрируйте свои рассуждения.

Совет: поощряйте разумные траты.

Покупки по-прежнему остаются одной из лучших возможностей для вашего ребенка попрактиковаться в математических концепциях, которые он изучает. Она может практиковать проценты и вычитание, вычисляя точную сумму, которую вы сэкономите, когда что-то поступит в продажу, и окончательную стоимость товаров со скидкой. Попросите ее помочь вам рассчитать чаевые, когда вы едите в ресторане. Если у нее есть сотовый телефон, ознакомьте его с деталями счета за сотовый телефон и размером платы за текст или за минуту использования, чтобы она могла научиться отслеживать, сколько она тратит.

Связанные

Длинное деление

Преобразуйте рациональные числа в десятичные с помощью длинного деления.

Пример:

Что такое 12/29 в виде десятичной дроби?

Что такое 13 4/5 в десятичном виде?

Выражения и уравнения

Создание простых уравнений

Используйте буквы для представления чисел в реальных математических задачах и генерируйте простые уравнения для их решения. Изобразите набор решений, если есть несколько ответов.

Пример:

Тесс, Нико и Сал собирают деньги на поездку в Стоунхендж. Тесс собрала T долларов, Нико собрал N долларов, а Сал собрал S долларов. Если Тесс собрала вдвое больше, чем Нико и Сал вместе взятые, соотношение можно выразить как T = 2 (N + S).

Решение для X

Определите значение переменной в уравнении и многоступенчатом уравнении.

Пример:

  • Решить относительно x: 5x + 6 = 46
  • Решить относительно b: 7 + 4b = 35
  • Решить относительно c: 2 (c + 7) = 26 + 10

Написание эквивалентных выражений

Использование диаграмм в качестве инструментов для понимания и создания эквивалентных математических выражений.

Геометрия

Масштаб

Используйте понимание пропорций и пропорций, чтобы понять масштаб: отношение длины на чертеже (или модели) объекта к длине реального объекта. В примерах проблемных рисунков масштаб от верхнего рисунка к нижнему составляет 1: 2 («один к двум»). Изменяйте масштаб и вычисляйте фактическую длину и площадь геометрических фигур.

Статистика и вероятность

Выборки

Понять концепцию случайной выборки и репрезентативного размера выборки.Используйте случайную выборку, чтобы делать выводы или заключения о совокупности из репрезентативной выборки.

Пример:

Репортер взял интервью у четырех новейших учителей в школьном округе города. Будет ли эта выборка репрезентативной?

Совет: Обсудите новости.

Когда вы вместе смотрите новости, следите за тем, как часто цитируется статистика. Обсудите детали любых упомянутых опросов. Обсудите, как используются эти концепции и какие моменты они используются для поддержки или опровержения.

Понимание вероятности

Понимайте вероятность как математическое представление вероятности того, что что-то, например событие или результат, произойдет. Большие числа представляют большую вероятность.

Рассчитайте шансы:

Если в вашей школе проводится лотерея, обсудите детали с вашим ребенком. Попросите его узнать, сколько билетов будет продано и сколько призов будет разыграно. Затем попросите его определить вашу вероятность выигрыша, если вы купите билет — 10 или 20.

Поощряйте понимание математики через спорт:

Спорт — это увлекательный способ изучения множества математических понятий. Любой заядлый фанат бейсбола знает, что игру нельзя по-настоящему оценить без понимания некоторых важных статистических данных, таких как средний результат игрока и количество забитых мячей. Футбол также полон статистических данных, таких как процент передач, выполненных квотербеком. . Если ваш ребенок увлечен спортом, предложите ему изучить его с помощью математики.

Расчет вероятности

Рассчитайте вероятность, разделив количество шансов на то, что событие или результат произойдет, на количество возможных исходов — например, если в сумке 10 апельсинов, 5 персиков и 15 яблок, вероятность случайного выбора персика — 5 из 30 (5/30 или 1/6). Рассчитайте вероятности простых и сложных событий.

Пример:

Какова вероятность выпадения шестерки с одного кубика? (простое событие)

Какова вероятность выпадения двойных шестерок с использованием двух кубиков (составное событие)

Буква должна быть выбрана из 26 букв английского алфавита.Какова вероятность выбора согласного? Объясните свои рассуждения.

Совет: математика на практике: разыграйте шансы.

Если в вашей школе проводится лотерея, обсудите детали с вашим ребенком. Попросите его узнать, сколько билетов будет продано и сколько призов будет разыграно. Затем попросите его определить вашу вероятность выигрыша — 10 или 20.

Чтобы узнать, как помочь семикласснику в классе математики, посетите нашу страницу с советами по математике для седьмого класса.

Ресурсы TODAY Parenting Guide были разработаны NBC News Learn с помощью профильных экспертов и соответствуют Общим основным государственным стандартам.

целей 7-го класса — проект Connected Mathematics

Goals 7 класса — Connected Mathematics Project Переключить специальные возможности
  1. Дом
  2. Математика
  3. Математические цели по единицам
  4. Цели 7-го класса

Формы и конструкции: двумерная геометрия

Свойства многоугольников:

Поймите свойства многоугольников, которые влияют на их форму.
  • Изучите способы сортировки многоугольников по семействам в соответствии с количеством и длиной их сторон, а также размером углов
  • Изучите закономерности между внутренними и внешними углами многоугольника
  • Изучите закономерности среди длин сторон многоугольника
  • Исследуйте симметрию вращения или отражения формы
  • Определите, какие многоугольники подходят друг к другу, чтобы покрыть плоскую поверхность и почему
  • Рассмотрение и решение проблем, связанных с различными полигонами

Отношения между углами:

Поймите особые отношения между углами.
  • Изучить методы оценки и измерения углов
  • Используйте инструменты для рисования углов
  • Причина о свойствах углов, образованных параллельными прямыми и поперечными линиями
  • Используйте информацию о дополнительных, дополнительных, вертикальных и смежных углах в форме для решения неизвестного угла в многоэтапной задаче

Построение многоугольников:

Поймите свойства, необходимые для построения многоугольников.
  • Нарисуйте или зарисуйте многоугольники с заданными условиями, используя различные инструменты и методы, такие как от руки, использование линейки и транспортира, а также использование технологий
  • Определите, при каких условиях будет создан уникальный многоугольник, более одного многоугольника или не будет многоугольника, особенно треугольников и четырехугольников.
  • Признать особые свойства многоугольников, такие как сумма углов, отношения длины сторон и симметрия, которые делают их полезными в строительстве, проектировании и природе
  • Решение задач, связанных со свойствами форм

Подчеркните негатив: целые и рациональные числа

Рациональные числа:

Развивайте понимание рациональных чисел, включая отрицательные рациональные числа.
  • Изучите отношения между положительными и отрицательными числами, моделируя их на числовой прямой
  • Используйте соответствующие обозначения для обозначения положительных и отрицательных чисел
  • Сравните и упорядочьте положительные и отрицательные рациональные числа (целые, дроби, десятичные дроби и ноль) и найдите их на числовой строке
  • Распознавать и использовать взаимосвязь между числом и его противоположностью (аддитивное обратное) для решения проблем
  • Отнесите направление и расстояние к числовой прямой
  • Используйте модели и рациональные числа для представления и решения проблем

Операции с рациональными числами:

Развивайте понимание операций с рациональными числами и их свойств.
  • Разработка и использование различных моделей (числовая линия, модель микросхемы) для представления сложения, вычитания, умножения и деления
  • Разработка алгоритмов сложения, вычитания, умножения и деления целых чисел
  • Интерпретируйте и напишите математические предложения, чтобы показать взаимосвязи и решить проблемы
  • Напишите и используйте связанные семейства фактов для сложения / вычитания и умножения / деления для решения простых уравнений
  • Используйте круглые скобки и порядок операций в вычислениях
  • Понять и использовать свойство коммутативности для сложения и умножения
  • Примените свойство распределенности для упрощения выражений и решения проблем

Растяжение и сжатие: понимание сходства

Подобные фигурки:

Поймите, что значит быть похожими на фигуры.
  • Найдите похожие фигуры, сравнив соответствующие стороны и углы
  • Используйте масштабные коэффициенты и соотношения для описания отношений между длинами сторон, периметрами и площадями аналогичных фигур
  • Обобщить свойства подобных фигур
  • Признать роль умножения в отношениях сходства
  • Признать взаимосвязь между масштабным коэффициентом и соотношением в аналогичных цифрах
  • Используйте неформальные методы, масштабные коэффициенты и геометрические инструменты для построения похожих фигур (масштабные чертежи)
  • Сравнить похожие цифры с не похожими цифрами
  • Различать алгебраические правила, дающие аналогичные фигуры, от правил, дающих несходные фигуры
  • Используйте алгебраические правила для получения похожих фигур
  • Распознавать, когда правило уменьшает или увеличивает фигуру
  • Изучите влияние на изображение фигуры, если число добавлено к x — или y -координатам вершин фигуры

Рассуждения с использованием похожих цифр Разработайте стратегии использования похожих фигур для решения задач.

  • Используйте свойства подобия, чтобы найти расстояния и высоты, которые нельзя измерить напрямую
  • Предсказать, как растяжение или сжатие фигуры повлияет на длину сторон, размеры углов, периметры и площади
  • Используйте масштабные коэффициенты или отношения, чтобы найти недостающие длины сторон в паре похожих фигур
  • Используйте сходство для решения реальных проблем

Сравнение и масштабирование: соотношения, скорости, проценты и пропорции

Коэффициенты, ставки и проценты:

Понимание соотношений, ставок и процентов.
  • Используйте соотношения, ставки, дроби и проценты для написания отчетов, сравнивающих две величины в данной ситуации
  • Различать и использовать в сравнениях как частичные, так и частичные отношения к целому
  • Используйте проценты для выражения соотношений и пропорций
  • Признать, что коэффициент — это особое соотношение, которое сравнивает два измерения с разными единицами
  • Анализировать сравнительные заключения, сделанные в отношении количественных данных, на предмет правильности и качества
  • Сделать суждение о том, какие сравнительные утверждения наиболее информативны или лучше всего отражают конкретную точку зрения в конкретной ситуации

Пропорциональность:

Поймите пропорциональность в таблицах, графиках и уравнениях.
  • Признайте, что постоянный рост в таблице, графике или уравнении связан с ситуациями пропорциональности
  • Напишите уравнение для представления шаблона в таблице или графике пропорционально связанных переменных
  • Свяжите единицу скорости и константу пропорциональности с уравнением, графиком или таблицей, описывающими ситуацию пропорциональности

Рассуждая пропорционально:

Разрабатывайте и используйте стратегии для решения проблем, требующих пропорционального рассуждения.
  • Распознавайте ситуации, в которых пропорциональное рассуждение подходит для решения проблемы
  • Масштабирование отношения, коэффициента, процента или дроби для сравнения или поиска эквивалентного представления
  • Используйте различные стратегии для поиска неизвестного в пропорции, включая масштабирование, таблицы ставок, процентные столбцы, удельные ставки и эквивалентные отношения
  • Установка и решение пропорций, возникающих из реальных приложений, таких как поиск скидок и наценок и преобразование единиц измерения

Движение вперед: линейные отношения

Линейные отношения:

Распознавайте проблемные ситуации, в которых две или более переменных имеют линейную связь друг с другом.
  • Определите и опишите закономерности изменения между независимыми и зависимыми переменными для линейных отношений, представленных таблицами, графиками, уравнениями или контекстными настройками
  • Создание таблиц, графиков и символьных уравнений, которые представляют линейные отношения
  • Определите скорость изменения между двумя переменными и интерцепциями x и y из графиков, таблиц и уравнений, которые представляют линейные отношения
  • Перевести информацию о линейных отношениях, данных в контексте, таблице, графике или уравнении, в одну из других форм
  • Напишите уравнения, которые представляют линейные отношения с учетом конкретной информации, и опишите, какую информацию представляют переменные и числа.
  • Установите связь между наклоном как отношением расстояния по вертикали к расстоянию по горизонтали между двумя точками на линии и скоростью изменения между двумя переменными, которые имеют линейную зависимость
  • Учтите, что y = mx представляет собой пропорциональную зависимость
  • Решать проблемы и принимать решения о линейных отношениях, используя информацию, представленную в таблицах, графиках и уравнениях

Эквивалентность:

Помните, что знак равенства указывает на то, что два выражения эквивалентны.
  • Помните, что уравнение y = mx + b представляет линейную зависимость и означает, что mx + b является выражением, эквивалентным y
  • Признайте, что линейные уравнения с одним неизвестным, k = mx + b или y = m ( t ) + b , где k , t , m , и b — постоянные числа, являются частными случаями уравнения y = mx + b
  • Признайте, что нахождение пропущенного значения одной из переменных в линейной зависимости, y = mx + b , аналогично поиску отсутствующей координаты точки ( x , y ), которая лежит на графике отношения
  • Решите линейные уравнения с одной переменной, используя символьные методы, таблицы и графики
  • Признать, что линейное неравенство с одним неизвестным связано с линейным уравнением
  • Решите линейные неравенства, используя графики или алгебраические рассуждения
  • Решите линейные неравенства, используя графики или алгебраические рассуждения
  • Написать и интерпретировать эквивалентные выражения

Чего вы ожидаете? Вероятность и ожидаемая стоимость

Экспериментальные и теоретические вероятности:

Изучите и изучите основные концепции вероятности и поймите, что вы можете строить вероятностные модели, собирая данные экспериментов (экспериментальная вероятность) и анализируя возможные равновероятные результаты (теоретическая вероятность).
  • Признать, что вероятности полезны для предсказания того, что произойдет в долгосрочной перспективе
  • Для события, описываемого повседневным языком, определите результаты в пространстве выборки, которые составляют событие
  • Интерпретировать экспериментальные и теоретические вероятности и взаимосвязь между ними и признать, что экспериментальные вероятности являются лучшими оценками теоретических вероятностей, когда они основаны на больших числах
  • Различать равновероятные и не равновероятные исходы путем сбора данных и анализа экспериментальных вероятностей
  • Поймите, что вероятность простых событий — это доля исходов в пространстве выборки, для которых событие происходит
  • Признать, что вероятность случайного события — это число от 0 до 1, которое выражает вероятность того, что событие произойдет
  • Оценить вероятность случайного события, собрав данные о случайном процессе, который его вызывает, и наблюдая его долгосрочную относительную частоту, и спрогнозировать приблизительную относительную частоту с учетом вероятности
  • Определить честность игры

Reasoning with Probability:

Изучите и разработайте вероятностные модели путем определения возможных результатов и анализа вероятностей для решения проблем.
  • Разработайте единую вероятностную модель, назначив равную вероятность всем исходам, и используйте модель для определения вероятностей событий
  • Разработайте вероятностную модель (которая может быть неоднородной), наблюдая частоты в данных, сгенерированных случайным процессом
  • Представляйте образцы пространств для простых и сложных событий и находите вероятности с помощью организованных списков, таблиц, древовидных диаграмм, моделей областей и моделирования
  • Поймите, что, как и в случае с простыми событиями, вероятность составного события — это доля исходов в пространстве выборки, для которых возникает составное событие
  • Разработайте и используйте моделирование для генерации частот для простых и сложных событий
  • Анализируйте ситуации, состоящие из двух этапов (или двух действий)
  • Используйте модели областей для анализа теоретических вероятностей двухэтапных результатов
  • Анализируйте ситуации с биномиальными исходами
  • Использование вероятности для расчета долгосрочного среднего значения азартной игры
  • Определить ожидаемое значение вероятностной ситуации
  • Используйте вероятность и математическое ожидание для принятия решения

Наполнение и упаковка: трехмерное измерение

Площадь поверхности и объем многоугольных призм и цилиндров: понимание площади поверхности и объема призм и цилиндров и их взаимосвязи.

  • Опишите призмы, используя их вершины, грани и ребра
  • Визуализируйте трехмерные формы и эффекты нарезания этих фигур плоскостями
  • Углубить понимание объема и площади поверхности прямоугольных призм
  • Оценить и рассчитать площадь поверхности и объем многоугольных призм, связав их с прямоугольными призмами
  • Изучите взаимосвязь между площадью поверхности и объемом призм
  • Соотношение площади поверхности и объема для общих цифр, особенно оптимизация площади поверхности для фиксированного объема
  • Предсказание влияния масштабирования размеров на линейные измерения, измерения площади и объема цилиндров и твердых фигур
  • Изучить взаимосвязь между объемом и площадью поверхности призм и цилиндров
  • Используйте объем и площадь поверхности призм, чтобы разработать формулы для объема и площади поверхности цилиндров
  • Откройте для себя, что объем призм и цилиндров можно рассчитать как произведение высоты и площади основания
  • Решение задач, связанных с площадью поверхности и объемом твердых фигур

Площадь и окружность кругов: понимание площади и окружности кругов и их взаимосвязи.

  • Связать площадь круга с охватом фигуры и окружность с окружающей фигурой
  • Оценить и вычислить площадь и длину окружности
  • Изучите взаимосвязь между радиусом (и диаметром) окружности и площадью
  • Изучите связь числа π с вычислением площади, оценив количество «квадратов радиуса», необходимых для покрытия круга
  • Исследуйте взаимосвязь между площадью и окружностью круга
  • Решение задач, связанных с площадью и окружностью окружностей

Объем сфер, конусов и пирамид: понимание взаимосвязи между объемом цилиндров и конусов, сфер и пирамид.

  • Отнести объем цилиндров к объему конусов, сфер
  • Оценить и рассчитать объем сфер, конусов и пирамид
  • Отнести объем конусов к объему цилиндров и объем пирамид к объему призм
  • Решать задачи, связанные с площадью поверхности и объемом сфер, конусов и пирамид

Выборки и популяции: сравнения и прогнозы

Процесс статистического исследования: понимание и использование процесса статистического исследования.

  • Задавайте вопросы, собирайте и анализируйте данные и делайте интерпретации, чтобы отвечать на вопросы
  • Применить Руководство по описанию распределений как инструмент, который будет использоваться на этапах анализа и интерпретации процесса статистического исследования
  • Создавайте и используйте простые опросы как метод сбора данных

Анализ распределений данных: понимание распределений данных и их анализа.

  • Различия данных и типы данных
  • Распознавание данных состоит из подсчетов или измерений переменной, которые называются распределением значений данных
  • Различать категориальные данные и числовые данные и определять, какие графики и статистические данные могут использоваться для представления каждого вида данных
  • Использовать несколько представлений
  • Организуйте и представляйте данные с помощью таблиц, точечных графиков, линейных графиков, столбчатых диаграмм значений, частотных столбчатых диаграмм, гистограмм и диаграмм типа «прямоугольник и усы»
  • Принимайте обоснованные решения о том, какие графики / таблицы используются для отображения анализируемых данных (привязка к заданным вопросам, типам данных и т. Д.))
  • Признайте, что данные, отображаемые с помощью графика, показывают общую форму распределения и дают общее представление о том, являются ли значения данных симметричными или несимметричными относительно центрального значения, или есть ли что-то необычное в форме
  • Признать, что можно вычислить одно число и использовать его для характеристики центра или того, что типично для распределения данных
  • Выявление и вычисление показателей центральной тенденции: среднего, медианного или режима данных
  • Определите, как медиана и среднее значение реагируют на изменения количества и величины значений данных в распределении
  • Принимайте информированные решения о том, какие меры центральной тенденции (среднее, медиана или мода) могут использоваться для описания распределения данных
  • Признать, что изменчивость возникает всякий раз, когда собираются данные, и описывать изменчивость в распределении данного набора данных
  • Опишите степень изменчивости в распределении, отметив, являются ли значения данных примерно одинаковыми или сильно разнесены
  • Выявление и вычисление показателей разброса: размах, межквартильный размах (IQR) и среднее абсолютное отклонение (MAD)
  • Разработать стратегии для анализа и / или сравнения распределений данных
  • Определите, какие статистические меры центра и разброса (среднее, медиана, мода, диапазон и т. Д.)) наиболее подходят для описания распределения данных
  • Используйте меры центра и разброса для сравнения распределений данных

Репрезентативные выборки: поймите, что статистику можно использовать для получения информации о совокупности путем изучения репрезентативной выборки населения.

  • Используйте случайную выборку для отбора репрезентативных выборок и используйте данные из этих выборок, чтобы делать выводы о популяциях
  • Изучить влияние размера выборки и процессов отбора выборки на показатели центра и изменчивости, которые описывают распределение выборки
  • Применить концепции вероятности для выбора случайных выборок из совокупностей
  • Сравните выборочные распределения, используя меры центра (среднее значение, медиана), меры разброса (диапазон, IQR, MAD) и данные, отображающие данные этой группы (гистограммы, диаграммы с квадратами и усами)

7 класс | Математика | Iowa Core

В 7 классе учебное время должно быть сосредоточено на четырех критических областях: (1) развитие понимания и применение пропорциональных отношений; (2) развитие понимания операций с рациональными числами и работа с выражениями и линейными уравнениями; (3) решение задач, связанных с масштабными чертежами и неформальными геометрическими конструкциями, и работа с двух- и трехмерными формами для решения задач, связанных с площадью, площадью поверхности и объемом; и (4) делать выводы о популяциях на основе выборок.

  1. Учащиеся расширяют свое понимание соотношений и развивают понимание пропорциональности для решения одно- и многоэтапных задач. Учащиеся используют свое понимание соотношений и пропорциональности для решения широкого круга задач, связанных с процентами, включая те, которые касаются скидок, процентов, налогов, чаевых, а также увеличения или уменьшения процента. Учащиеся решают задачи о масштабных чертежах, соотнося соответствующие длины между объектами или используя тот факт, что отношения длин внутри объекта сохраняются в подобных объектах.Учащиеся составляют график пропорциональных соотношений и неформально понимают единичную ставку как меру крутизны соответствующей линии, называемой наклоном. Они отличают пропорциональные отношения от других отношений.
  2. Учащиеся развивают единое понимание чисел, распознавая дроби, десятичные дроби (которые имеют конечное или повторяющееся десятичное представление) и проценты как различные представления рациональных чисел. Учащиеся распространяют сложение, вычитание, умножение и деление на все рациональные числа, сохраняя свойства операций и отношения между сложением и вычитанием, а также умножением и делением.Применяя эти свойства и рассматривая отрицательные числа с точки зрения повседневного контекста (например, суммы задолженности или температуры ниже нуля), учащиеся объясняют и интерпретируют правила сложения, вычитания, умножения и деления с отрицательными числами. Они используют арифметику рациональных чисел, когда формулируют выражения и уравнения с одной переменной и используют эти уравнения для решения задач.
  3. Ученики продолжают свою работу с площадью с 6 класса, решая задачи, связанные с площадью и окружностью круга и площадью поверхности трехмерных объектов.Готовясь к работе над соответствием и сходством в 8-м классе, они рассуждают о взаимосвязях между двумерными фигурами, используя масштабные чертежи и неформальные геометрические конструкции, и они знакомятся со связями между углами, образованными пересекающимися линиями. Учащиеся работают с трехмерными фигурами, связывая их с двухмерными фигурами, исследуя поперечные сечения. Они решают реальные и математические задачи, касающиеся площади, площади поверхности и объема двух- и трехмерных объектов, состоящих из треугольников, четырехугольников, многоугольников, кубов и прямых призм.
  4. Учащиеся опираются на свою предыдущую работу с отдельными распределениями данных, чтобы сравнить два распределения данных и ответить на вопросы о различиях между популяциями. Они начинают неформальную работу со случайной выборки для создания наборов данных и узнают о важности репрезентативных выборок для составления выводов.

Решение алгебраических уравнений: определение и примеры — математический класс [видео 2021 года]

Немного базовой терминологии

Математика с буквами — это просто расширение математики без букв.Алгебра просто упрощает разговор о чем-то с неизвестной ценностью, и вам не нужно делать сумасшедшие утверждения, как мы только что сделали.

Математики согласились называть букву, которая используется для обозначения неизвестной величины, переменной . Чтобы сбить с толку, он называется переменной, даже если представляет собой одно конкретное число, как в случае с нашим примером уравнения. Пять — единственное число, которое делает равенство 3 x + 2 = 17 истинным. Но даже после того, как вы это узнаете, x по-прежнему называется переменной.

3 из 3 x + 2 = 17 называется коэффициентом, а 2 и 17 называются константами; мы можем называть их постоянными членами. Любые термины, умноженные на одну и ту же переменную или комбинацию переменных, подобны терминам. 3 y и 10 y похожи на термины, как и 3 xy и 17.23 xy . Сравните их с 3 x и 7 y , которые не похожи на термины и не могут быть объединены.

Теперь, когда мы разобрались с этим, давайте разберемся с алгебраическими уравнениями.

Алгебраическое уравнение: определение

Есть несколько правил, которые мы должны соблюдать:

  • Алгебраическое уравнение должно содержать переменную.
  • Переменная должна быть умножена на коэффициент, отличный от нуля.
  • Должен быть знак равенства.

Является ли наше уравнение 3 x + 2 = 17 алгебраическим уравнением?

Да! Он имеет переменную, умноженную на ненулевой коэффициент (3), и имеет знак равенства, поэтому он соответствует нашим требованиям.

Решение уравнений с одной переменной

Решение алгебраического уравнения просто означает манипулирование уравнением так, чтобы переменная сама по себе находилась на одной стороне уравнения, а все остальное — на другой стороне уравнения. Как только все остальное упростится, уравнение решено.

Самым простым алгебраическим уравнением, которое вы могли бы иметь, было бы что-то вроде x = 5, которое одновременно является алгебраическим уравнением и собственным решением.

Давайте попробуем что-нибудь посложнее: y + 5 = 10.

Как мы можем получить y отдельно? Да ну избавиться от 5 конечно! Только не все так просто. Стороны уравнения во многом похожи на братьев и сестер: если вы сделаете что-то для одного, а не для другого, кто-то начнет кричать: « Это несправедливо! ». Чтобы избежать этой ситуации, что бы мы ни делали с одной стороной уравнения, нам нужно делать и с другим. Что нам нужно сделать с левой стороны, чтобы избавиться от надоедливой пятерки?

Вычтем 5 из обеих частей уравнения. Это превращает наше уравнение в следующее:

y + 5-5 = 10-5

Это немного неуклюже, поэтому давайте объединим одинаковые термины:

y + (5-5) = (10-5) )

5-5 = 0 и 10-5 = 5, поэтому наше уравнение принимает следующий вид:

y = 5

Теперь это решено! По мере того, как вы ближе познакомитесь с этими типами операций, вы можете пропустить промежуточные шаги и просто перейти от y + 5 = 10 к y = 5 за один шаг.А пока вам следует выписать каждый шаг. Это хорошая практика, которая также помогает вашим учителям понять, с какими шагами у вас возникают проблемы.

Еще один совет: не думайте, что вы знаете, сколько места вам понадобится для решения уравнения. Это часто приводит к беспорядку, поэтому избегайте этого! Оставьте много бумаги для выработки каждого решения, чтобы у вас никогда не закончилось место. Еще лучше не записывать ничего для следующей задачи, пока не закончите ту, над которой работаете.

Дополнительная практика

Тот же процесс, который мы видели ранее (перемещение за исключением переменной в другую часть уравнения), работает независимо от того, какая операция требуется.

Давайте решим наше исходное уравнение: 3 x + 2 = 17. Как вы думаете, будет легче сначала избавиться от 3 или 2? Хорошая новость в том, что вы можете делать это в любом порядке. Начнем с 3:

(3 x + 2) / 3 = 17/3

Это сокращается до:

x + 2/3 = 17/3

О боже, вероятно, было бы было лучше начать с 2. Ну что ж, давайте продолжим:

x + 2/3 — 2/3 = 17/3 — 2/3

Теперь объедините похожие термины:

x = 15 / 3

И, наконец:

x = 5

Почему бы вам не попробовать ту же задачу, но начать с манипулирования 2 вместо 3.Посмотрим, сможете ли вы придумать такой же ответ! Возможно, вам будет легче, чем то, что мы только что сделали.

Когда мы начинаем говорить о переменных с показателями степени или уравнениях с несколькими переменными, решения могут стать немного более сложными. Однако вы должны быть рады узнать, что все правила и методы, описанные в этом уроке, по-прежнему применимы к этим более сложным задачам. Язык математики строится сам на себе. Разве математика не прекрасна?

Резюме урока

Хорошо, давайте сделаем пару минут, чтобы повторить.Как мы узнали на этом уроке, алгебраическое уравнение состоит из переменной, ненулевого коэффициента и констант. И помните, что переменная — это просто буква, которая используется для обозначения неизвестной величины.

Решение этого типа уравнения включает в себя манипулирование им в соответствии с логическими математическими правилами, так что вы можете найти нужную переменную, выделив ее с одной стороны уравнения, а все остальное с другой. Представьте, что каждая сторона равенства — это ребенок: что бы вы ни делали с одной стороной, вы должны сделать и с другой стороной.Как только вы усвоите эти концепции, решение алгебраических уравнений станет легким делом!

Алгебраические выражения — объяснения и примеры

Алгебра — интересный и увлекательный раздел математики, в котором числа, фигуры и буквы используются для выражения задач. Независимо от того, изучаете ли вы алгебру в школе или сдаете какой-то тест, вы заметите, что почти все математические задачи представлены словами.

Следовательно, необходимость переводить письменные текстовые задачи в алгебраические выражения возникает тогда, когда нам нужно их решить.

Большинство алгебраических задач со словами состоят из рассказов или примеров из реальной жизни. Другие — простые фразы, такие как описание математической задачи. В этой статье вы узнаете, как написать алгебраических выражения из простых текстовых задач, а затем перейти к слегка сложным текстовым задачам.

Что такое алгебраическое выражение?

Многие люди попеременно используют алгебраические выражения и алгебраические уравнения, не подозревая, что это совершенно разные термины.

Алгебраика — это математическая фраза, в которой две стороны фразы соединены знаком равенства (=). Например, 3x + 5 = 20 — это алгебраическое уравнение, где 20 представляет собой правую часть (RHS), а 3x +5 представляет собой левую часть (LHS) уравнения.

С другой стороны, алгебраическое выражение — это математическая фраза, в которой переменные и константы объединяются с помощью операционных символов (+, -, × & ÷). В алгебраическом символе отсутствует знак равенства (=). Например, 10x + 63 и 5x — 3 являются примерами алгебраических выражений.

Давайте рассмотрим терминологию, используемую в алгебраических выражениях:

  • Переменная — это буква, значение которой нам неизвестно. Например, x — это наша переменная в выражении: 10x + 63.
  • Коэффициент — это числовое значение, используемое вместе с переменной. Например, 10 — это переменная в выражении 10x + 63.
  • Константа — это терм, который имеет определенное значение. В этом случае 63 — это константа в алгебраическом выражении 10x + 63.

Существует несколько типов алгебраических выражений, но основной тип включает:

  • Мономиальное алгебраическое выражение

Этот тип выражения имеет только один член, например, 2x, 5x 2 , 3xy и т. Д. .

Алгебраическое выражение, имеющее два отличных члена, например 5y + 8, y + 5, 6y 3 + 4 и т. Д.

Это алгебраическое выражение с более чем одним членом и ненулевыми показателями. переменных.Примером полиномиального выражения является ab + bc + ca и т. Д.

Другие типы алгебраических выражений:

Числовое выражение состоит только из чисел и операторов. В числовое выражение переменная не добавляется. Примеры числовых выражений: 2 + 4, 5-1, 400 + 600 и т. Д.

Это выражение содержит переменные вместе с числами, например, 6x + y, 7xy + 6 и т. Д.

Как решить алгебраическое выражение?

Цель решения алгебраического выражения в уравнении — найти неизвестную переменную.Когда два выражения приравниваются, они образуют уравнение, и поэтому становится легче найти неизвестные члены.

Чтобы решить уравнение, поместите переменные с одной стороны, а константы — с другой. Вы можете изолировать переменные, применяя арифметические операции, такие как сложение, вычитание, умножение, деление, квадратный корень, кубический корень и т. Д.

Алгебраические выражения всегда взаимозаменяемы. Это означает, что вы можете переписать уравнение, поменяв местами LHS и RHS.

Пример 1

Вычислите значение x в следующем уравнении

5x + 10 = 50

Решение

Учитывая уравнение 5x + 10 = 50

    Изолируйте переменные и константы;
  • Вы можете сохранить переменную на левой стороне, а константы на правой.

5x = 50-10

5x = 40

Разделим обе части на коэффициент переменной;

x = 40/5 = 8

Следовательно, значение x равно 8.

Пример 2

Найдите значение y, когда 5y + 45 = 100

Решение

Изолировать переменные от констант;

5y = 100 -45

5y = 55

Разделим обе части на коэффициент;

y = 55/5

y = 11

Пример 3

Определите значение переменной в следующем уравнении:

2x + 40 = 30

Решение

Разделите переменные из константы;

2x = 30-40

2x = -10

Разделите обе стороны на 2;

x = -5

Пример 4

Найдите t, когда 6t + 5 = 3

Решение

Отделите константы от переменной,

5 —0002 6t 3

6t = -2

Разделим обе части на коэффициент,

t = -2/6

Упростим дробь,

t = -1/3

Практические вопросы

1.

Добавить комментарий

Ваш адрес email не будет опубликован. Обязательные поля помечены *